Download docx - Cardiology question

Transcript
Page 1: Cardiology question

A 68-year-old woman recently diagnosed with multiple myeloma presents to her GP with progressively increasing breathlessness, exercise intolerance and ankle swelling. On examination, there is bilateral pitting leg oedema to her thighs, ascites and raised JVP. The apical impulse is impalpable. An ECG shows diffusely diminished voltage. Chest X-ray is normal and the echocardiogram shows small thick ventricles and dilated atria with a thickened interatrial septum. The ventricular myocardium has a granular sparkling texture on echo, and minimal fluid in the pericardial space is noted.

What is the most likely diagnosis? Chronic pericardial effusion with tamponade Chronic pericardial effusion without tamponade Constrictive pericarditis Restrictive cardiomyopathy Congestive heart failure

Cardiac involvement is the most common cause of death in patients with amyloidosis associated with an immunocyte dyscrasia – typically as restrictive cardiomyopathy. Physical examination reveals right heart failure with a raised jugular venous pressure (JVP), characteristically showing a prominent deep Y descent. The heart size is often normal. The physical findings are very similar in constrictive pericarditis (CCP), but the apex is frequently non-palpable due to the thick pericardium. The chest X-ray may show pericardial calcifications in patients with constrictive pericarditis. The most characteristic ECG finding of restrictive cardiomyopathy is diffusely diminished voltages. Echocardiography typically shows small thick ventricles and a thick interatrial septum due to amyloid deposits, which have a ‘granular sparkling’ appearance. Pericardial effusion is common, but rarely causes tamponade.

A 58-year-old man’s ECG shows a combination of a prolonged QT interval with tall T waves.

What is this suggestive of?Uraemia Hypocalcaemia Hypokalaemia Hypermagnesaemia Metabolic alkalosis

A prolonged QT interval is due to hypocalcaemia and tall T waves to hyperkalaemia and/or acidosis, which can be caused by uraemia. The main ECG change resulting from hypocalcaemia is a long QT interval due to prolongation of the ST segment.Hypokalaemia is caused by potassium-wasting diuretics, potassium-wasting diarrhoea and hypokalaemic periodic paralysis. The hallmark of the effect of hypokalaemia on the ECG is the development of large U waves (positive deflection after the T wave). The normal U wave is produced by repolarisation of the His–Purkinje system.

A 32-year-old man is brought to A&E in a collapsed state having sustained a precordial stab wound.

Which of the following cardiac valves is most likely to have been injured?Aortic valve Tricuspid valve Pulmonary valve Mitral valve Thebasian valve

The tricuspid valve is the most anterior valve of the human heart and is the commonest to be injured during a stabbing attack. Penetrating injuries may cause lacerations to any of the heart chambers or great vessels, and death may result from haemorrhage or cardiac tamponade. Late complications include

1

Page 2: Cardiology question

infective pericarditis, valve damage or intracardiac shunts. Echocardiography is useful in diagnosing the underlying problem.

A 30-year-old woman with a previous history of deep vein thrombosis is expecting her first child.

During which phase of her pregnancy and puerperium does she have the greatest risk of venous thrombosis?First trimester Second trimester Third trimester During delivery First 6 weeks after delivery

There is an increase in thromboembolic complications because of the hypercoagulability that exists postpartum. Anticoagulants may be necessary during pregnancy to prevent or control the following: venous thrombosis, pulmonary embolism, rheumatic mitral valve disease, prosthetic heart valves, peripartum cardiomyopathy, primary pulmonary hypertension and Eisenmenger’s syndrome.

A 28-year-old woman who is known to have a cardiac murmur becomes pregnant. It is noted that the intensity of her murmur diminishes during her pregnancy.

Which cardiac abnormality is she likely to have?Aortic stenosis Aortic regurgitation Mitral stenosis Pulmonic stenosis Fallot’s tetralogy

During pregnancy, cardiac output and blood volume increase from the second month up to the thirtieth week to 30–50% above the normal levels. The average increase in blood volume during pregnancy amounts to 1600 ml, and there is also an increased metabolic workload. This produces warm extremities, a tachycardia with a large-volume pulse and a slight rise in venous pressure. The diastolic blood pressure is lower due to vasodilatation, and this is responsible for the fading of the aortic regurgitation murmur. The apex beat is displaced, because of cardiomegaly and a raised diaphragm. The increased blood flow may produce a pulmonary systolic murmur and a third heart sound. All stenotic murmurs become more prominent.

A 58-year-old-woman suffers a cardiac arrest while on the ward. A rhythm strip shows VF.

What is the strength (in joules) recommended for the monophasic shock used for defibrillation?50J100J200J300J360J

Three-quarters of arrests are due to ventricular fibrillation. Only a small proportion is due to pulseless electrical activity (PEA), the rest being due to asystole. PEA may have a potentially reversible cause:•        hypovolaemia•        hypoxia•        hyperkalaemia•        hypokalaemia

2

Page 3: Cardiology question

•        hypothermia•        tension pneumothorax•        tamponade•        toxicity due to drugs•        thromboembolism

Defibrillation is used to convert VF to sinus rhythm. Previously the recommendation was Initially a monophasic 200-joule shock, followed by 200-J and then 360-J shocks. New resusciation guidelines now recommend shocking at 360-J.

A 30-year-old-man presents to the outpatient clinic with a 2-month history of progressive effort intolerance. Some three weeks ago he experienced an episode of shortness of breath at rest, suggestive of paroxysmal nocturnal dyspnoea. Examination reveals a JVP raised up to his earlobes, a soft tender hepatomegaly and a bilateral pitting oedema up to his knees. Chest examination reveals bibasal crepitations, and an audible S3 on auscultation of the heart. The chest X-ray shows cardiomegaly with interstitial infiltrates. Echocardiography shows global left ventricular hypokinesia with an ejection fraction of 25–30%.

Which of the following is the LEAST likely aetiological factor?Alcohol abuseGenetic factorAdenovirusEosinophilic statesHIV infection

Dilated cardiomyopathy encompasses a heterogeneous group of conditions. Alcohol abuse is an important aetiological factor in a significant number of patients. About 25% of cases are inherited as an autosomal-dominant trait. A substantial group is due to a late autoimmune reaction to viral myocarditis. Up to 10% of patients with advanced HIV infection develop dilated cardiomyopathy. Eosinophilic states are associated with obliterative cardiomyopathy.

Which of the following statements is true regarding pulsus alternans?It is found in beriberi heart diseaseThe pulse is irregularIt is diagnosed electrocardiographicallyIt is found in association with a third heart soundIt is found in patients with pericardial effusion

Pulsus alternans is found in patients with acute left ventricular failure. Alternate weak and strong pulses occur, which are regularly placed. A pathological third heart sound is usually associated.The condition may be associated with heart failure or pericarditis for example, but is not usually seen in association with pericardial effusion. Beri-Beri heart disease leads to long QT, T wave inversion and low voltage complexes.Electrical alternans is diagnosed electrocardiographically. The amplitude of QRS complexes varies alternately. It occurs due to changes in electrical depolarisation,  conduction abnormalities or cardiac motion.

Which of the following is the commonest cardiovascular abnormality seen in an adult patient with Marfan’s syndrome?

Aortic regurgitationAortic root dilatationMitral regurgitationMitral annular calcificationAortic dissection

3

Page 4: Cardiology question

Marfan’s syndrome is a connective tissue disorder that is inherited as an autosomal-dominant trait. There is considerable variation in its clinical manifestations. The ocular (dislocation of the lens), skeletal (arachnodactyly, joint hypermobility, scoliosis, chest deformity and high arched palate) and cardiovascular systems (aortic root dilatation (70%) and mitral valve prolapse (60%)) are characteristically involved. Weakening of the aortic media leads to a fusiform ascending aortic aneurysm, which may be complicated by aortic regurgitation and aortic dissection.Mitral regurgitation can result from mitral valve prolapse, dilatation of a mitral valve annulus or mitral annular calcification. Pregnancy is particularly hazardous. Treatment with ß-blockers reduces the rate of aortic dilatation and the risk of rupture.

An elderly man is admitted to the ICU and put on intermittent positive-pressure ventilation.

Which of the following statements is true when compared to spontaneous ventilation?Lung volumes are decreasedPulmonary vascular resistance is decreasedSystemic blood pressure risesVenous return and cardiac output fallIntrathoracic pressure is decreased

During intermittent positive-pressure ventilation (IPPV), lung volumes are significantly increased when compared to spontaneous ventilation. A large tidal volume causes a rise in pulmonary vascular resistance, which may lead to pulmonary hypertension and right ventricular compromise. The over-inflated alveoli cause compression of the alveolar blood vessels. Moreover, the resultant increase in RV volume may impede LV filling (ventricular interdependence). Hyperinflation also releases prostaglandins, which decrease blood pressure. The intrathoracic pressure is increased at all points in the respiratory cycle.Inspiration during IPPV increases intrathoracic pressure and so increases right atrial pressure relative to atmospheric pressure, therefore leading to decreased venous return. The increased intrathoracic pressure also decreases the gradient across the LV that it has to work against, which results in a decreased afterload. Both these effects reduce intrathoracic blood volume. 

A 22-year-old-woman presents to A&E with a 4-day history of chest pain. She has been unwell with an influenza-like illness for the last week. The ECG shows widespread ST elevation in the inferior, anterior and lateral leads.

What ECG changes would you expect to see in the next week or two?Development of deep Q waves in all leadsST depression in inferior and lateral leadsT-wave inversion in all leadsTall and peaked T waves in all leadsLoss of R waves in all leads

The ECG changes in acute pericarditis consist of ST elevation with concavity upward in all leads facing the epicardial surface, ie anterior, inferior and lateral. Only ‘cavity’ leads avr, V1 and, rarely, V2 show ST depression. This is followed by the return of ST segments to baseline and flat T waves. T waves then become inverted without the loss of R waves or development of Q waves. As the illness improves, T waves become normal but may occasionally persist in patients with chronic pericarditis. Rhythm and conduction abnormalities are not typical in pericarditis unless the myocardium is involved.Causes of pericarditis:•        Acute idiopathic pericarditis•        Infections: viral infections (eg coxsackievirus B)

tuberculosisother bacteriafungi

•        Inflammatory; post-MI/cardiotomy: autoimmune rheumatic disorder

4

Page 5: Cardiology question

•        Others: neoplasticuraemiatraumaaortic dissectionhypothyroidismirradiationdrugs, eg hydralazine

Which of the following statements is true of raised cardiac troponin levels in the blood?

Are commonly seen after DC cardioversionRemain elevated for up to two days after myocardial damageAre seen in patients with NSTEMICan be used to distinguish Non-Q from Q MICan be found in patients with hypertrophic cardiomyopathy

The most sensitive markers of myocardial cell damage are the cardiac troponins T and I. Troponins are regulatory elements of the contractile apparatus in muscle. They are released within 4–6 hours and remain elevated for up to two weeks.  Unstable angina and ST segment myocardial infarction are different ends of the spectrum of myocardial damage, with greater rises in troponin seen towards the more severe end of the spectrum. Angina associated with a troponin rise is essentially an NSTEMI. Cardioversion can give rise to raised creatine kinase (CK) levels due to skeletal muscle damage. Non-Q and Q MI are electrocardiographic diagnoses.Raised troponin levels indicate heart muscle damage: the commonest cause being ischaemic damage. Myocarditis or myocardial contusion can also cause raised troponin levels.

A 32-year-old-woman was cross-country skiing when she fell down a water-filled gully and became trapped beneath an ice-sheet. Frantic efforts were made to extract her, but after 40 minutes all movements ceased.

Which of the following statements is true?Her pulseless state is, in all likelihood, due to ventricular fibrillationDefibrillation at the scene is likely to succeedMetabolic alkalosis will quickly set inAspirated seawater is more likely to produce pulmonary oedema than fresh waterIt is important to lift her out of water in the prone position

The management of patients who nearly drown in cold water is quite different from that for routine cardiopulmonary arrests. Re-warming such patients should be undertaken in a hospital that has extracorporeal re-warming facilities. Head-out upright immersion in water at body temperature results in a 32–66% increase in cardiac output because of the pressure of the surrounding water. Resistance to circulation is suddenly removed as the person leaves the water, which when added to venous pooling can cause circulatory collapse. This is believed to be the cause of death in many individuals. To counter this effect, patients should be lifted out of the water in the prone position. Continuous chest compression should be applied throughout transportation. This alone is as effective as chest compression with expired air resuscitation. Because hypothermia may render the carotid pulse impalpable, it is important to commence chest compression with firm evidence of cardiac arrest. Electrocardiographic monitoring should be available. Defibrillation is ineffective if the myocardium is cold. Evidence suggests that aspirated fresh water is more likely than seawater to produce pulmonary oedema. Metabolic acidosis can develop and should be corrected with adequate oxygenation and plasma expansion.

A 79 year-old-man known to have chronic congestive heart failure is readmitted with worsening heart failure. His furosemide (frusemide) dosage is increased to 200 mg/day to aid the relief of his symptoms. His other medications are bendrofluazide, ramipril and bisoprolol.

5

Page 6: Cardiology question

Which of the following effects can be encountered?HyperkalaemiaHypercalciuriaHypermagnesaemiaHyperuricaemiaHypoalbuminaemia

Diuretics can cause various electrolyte disturbances, eg hyponatraemia, hypomagnesaemia and hypokalaemia.Metabolic alkalosis results from chloride loss and extracellular fluid volume contraction. Extracellular volume depletion and raised urea concentrations can occur due to renal hypoperfusion. Typically, however, serum urea concentrations are unchanged or do not increase by more than 10–20% of baseline values.Tubular handling of uric acid is complex, with both reabsorption and secretion occurring in the proximal tubule. Diuretics can interfere with either of these processes thereby causing hyperuricaemia. The effect is usually dose-dependent and frequently asymptomatic. Clinical gout is more likely if the patient is also extracellular fluid volume-depleted.Ototoxicity is an uncommon side-effect occurring with high-dose loop diuretic therapy. It has been noted in patients with renal failure who are given high-dose (> 2 g/day) infusion therapy.

A 68-year-old-man with atrial fibrillation (AF) is admitted electively for DC cardioversion, to be performed as a day-case procedure. However, the procedure is postponed to a later date.

Which one of the following reasons could be responsible for the delay?He had discontinued digoxin for the last 2 days He was taking amiodarone His INR 3 weeks ago was 1.6 His serum potassium level was 4.2 mEq/l He had an episode of angina 2 days ago

Although the INR on the day of cardioversion is important, the INR should be optimal in the preceding 3-4 weeks prior to cardioversion. External cardioversion is a safe and effective method for restoring sinus rhythm, and should be attempted at least once in every patient with chronic AF. Overt congestive heart failure, hypokalaemia and hypothyroidism should be controlled as much as possible before cardioversion. Acute myocardial infarction is not a contraindication to cardioversion. If the patient has a slow ventricular response of AF in the absence of anti-arrhythmic drugs, cardioversion should be performed after the insertion of a temporary transvenous-pacing catheter.Electrical cardioversion is initially successful in 70–94% cases, but relapse is frequent (25–50% at 1 month and 70–90% at 1 year). The success depends on the duration of AF, transthoracic impedance, left atrial size and the age of the patient.Digoxin should be withheld on the day of cardioversion. However, if digoxin toxicity is suspected, the problem should be resolved before cardioversion is attempted. Pre-treatment with amiodarone or sotalol may prevent early recurrence.The initial shock strength should be 100 J, followed by a second 200-J shock and a third 360-J shock. If AF persists, a second 360-J shock with the paddles in the anteroposterior position can be attempted. Immediate DC cardioversion, after the administration of intravenous heparin, is appropriate in an emergency or if AF has been present for less than 48 hours. In elective cases, patients should be established on warfarin to give an INR of between 2 and 3 for a minimum of three weeks prior to cardioversion. Anticoagulation should be continued for four weeks after successful cardioversion.

Which of the following is a characteristic feature of troponin? It is an integral component of pericardial cells Levels rise immediately or even prior to the onset of chest pain due to myocardial infarction About 30% of infarct patients show a rise in levels at 12 hours from the onset of symptoms

6

Page 7: Cardiology question

A level > 1 ng/ml indicates myocardial infarction Levels act as a prognostic factor following an acute coronary syndrome

The troponin complex is part of the cardiac myofibril and is released in myocardial damage. Levels rise about 4 hours after the onset of chest pain. 100% of patients are positive for troponin at 12 hours after the onset of pain. A level of > 0.1 ng/ml is considered as a significant rise. Levels of troponin have a strong relationship to clinical outcomes, such as progression to myocardial infarction and death.

A young computer programmer suddenly develops dysphasia and right-sided weakness. Cardiac examination is normal and he is afebrile.

Which investigation would confirm the underlying cardiological diagnosis? Chest X-ray 12-lead ECG 2-D echocardiography Carotid Doppler study Transoesophageal echocardiogram

A young, otherwise healthy person who suddenly develops a stroke is likely to have a paradoxical embolism due to a patent foramen ovale (PFO or ostium secundum defect). PFOs are relatively common and may be present in 30% of the general population. PFOs greater than 4 mm and associated with shunting are more likely to be associated with systemic emboli. A chest X-ray may show enlargement of the heart and pulmonary artery as well as pulmonary plethora. Incomplete right bundle branch block is seen on ECG. Echocardiography may demonstrate the defect and show right ventricular dilatation and hypertrophy and pulmonary artery dilatation. However, PFOs may be missed on 2-D echocardiogram. The precise size and location of the defect can be shown on transoesophageal echocardiography. A transoesophageal echocardiogram with Doppler colour-flow imaging would be the investigation of choice in this case.

Right ventricular myocardial infarction is characterised by which of the following?

ST-segment elevation in leads II, III and aVF with Q waves and T-wave inversion in these leads Occlusion of the left coronary artery Marked pulmonary vascular congestion A rise in systolic blood pressure Absent Kussmaul’s sign

Right ventricular myocardial infarction usually occurs in association with an inferior-wall left ventricular infarction, as revealed by the ECG. There is usually a right coronary occlusion. Characteristic clinical features include a low cardiac output syndrome with jugular venous distension but no pulmonary vascular congestion. Kussmaul’s sign (increased jugular venous distension with inspiration) may be evident.

A 70-year-old woman is admitted with chest pain and breathlessness. On examination, her heart rate is 170 beats/min, her BP is 125/72 mmHg. ECG shows atrial fibrillation.

What is the next step in her management?Administration of propranololAdministration of verapamilAsynchronous cardioversionAdministration of warfarinImmediate heparinisation

7

Page 8: Cardiology question

The appropriate treatment for atrial fibrillation (AF) is determined by the patient’s relative risk from the arrhythmia. High-risk patients include those with a heart rate greater than 150 beats/min, chest pain, an unstable condition or shock. These patients require urgent treatment. Immediate heparinisation, to reduce the risk of systemic embolisation, and attempted cardioversion with synchronised DC shock should be carried out first. Warfarin treatment is indicated in the elderly and those with heart disease. Young patients with lone atrial fibrillation in the absence of heart disease may not need anticoagulation.

A 75-year-old man with congestive cardiac failure presents with atrial fibrillation. He is haemodynamically stable with a ventricular rate of 72. He has a good functional state, although ECHO cardiography revealed a dilated left atrium and mild mitral regurgitation.

Which drug option would be most beneficial for this patient?AspirinDigoxinFrusemideLidocaineWarfarin

Patients with atrial fibrillation who are stable pose an intermediate risk. The initial treatment in this case is anticoagulation with warfarin. This is indicated in valvular heart disease and in the elderly. Digoxin is effective in controlling the heart rate at rest, and may improve his symptoms but would not affect stroke risk.

A 54-year-old man suddenly develops weakness of the left side of his face and arm and difficulty in speech. This episode lasts for 15 minutes. He has a history of hypertension, which is well controlled on a calcium channel blocking agent. His brother had had a severe disabling stroke at the age of 50.Cholesterol level is 5.8mmol/l. CT scan performed the same day shows the presence of 2 old lacunar strokes in the right middle cerebral artery territory. CT angiogram of the carotid system shows a 60% stenosis of the right internal carotid artery.

Which of the following factors is the strongest predictor of his being at a high risk of early recurrent stroke?Positive family history History of hypertension Hyperlipidaemia Presence of moderate carotid stenosis Presence of previous strokes on CT scan

This is a transient ischaemic attack (TIA). About 15–20% of patients with stroke have a preceding transient ischaemic attack. The issue of subsequent stroke prevention is therefore paramount when managing such a “warning event”. The urgency of treatment of minor stroke or transient ischaemic attack should depend on the early risk of major stroke. The risk of recurrent stroke during the first few days after a transient ischaemic attack or minor stroke is much higher than previously estimated. Recent studies have identified potential risk factors for those at highest risk of subsequent stroke: age>60 years; hypertension; duration of symptoms >60 minutes; certain clinical features (unilateral weakness, speech impairment); presence of diabetes mellitus. Brain imaging also seems to be of prognostic value: the presence of infarction on CT brain scans in patients with transient ischaemic attack or minor stroke is associated with an increased risk of stroke recurrence. More research is needed to determine the optimal medical management according to individual risk factors. There are several treatments that are likely to be effective in preventing stroke in the acute phase after a transient ischaemic attack or minor ischaemic stroke including aspirin, possibly in combination with clopidogrel and anticoagulation in patients with atrial fibrillation, and possibly statins.

8

Page 9: Cardiology question

The subgroup of patients with large-artery atherosclerosis (usually carotid bifurcation stenosis) accounts for the largest proportion of early recurrent strokes. A recent population-based study of prognosis of patients with transient ischaemic attack and =50% symptomatic carotid-artery stenosis reported risks of stroke of about 20% during the 2 weeks before endarterectomy and other studies have highlighted the high risk of stroke if endarterectomy is delayed, and hence the rapid decrease in benefit from surgery with increasing time since event. For neurologically stable patients with transient ischaemic attack and minor stroke, benefit from endarterectomy is greatest if done within 2 weeks of the event. NB. The risk benefit ratio of treating symptomatic carotid stenosis (secondary stroke prevention) differs from that of treating asymptomatic stenosis as part of primary prevention, where stenosis must be severe in order to justify the risk of surgery.

From the American Heart Association guidelines (2006):

For patients with recent TIA or ischemic stroke within the last 6 months and ipsilateral severe (70 to 99%) carotid artery stenosis, CEA is recommended by a surgeon with a perioperative morbidity and mortality of <6%

Class I, Level A

For patients with recent TIA or ischemic stroke and ipsilateral moderate (50 to 69%) carotid stenosis, CEA is recommended, depending on patient-specific factors such as age, gender, comorbidities, and severity of initial symptoms.

Class I, Level A

When degree of stenosis is <50%, there is no indication for CEAClass III, Level A

A patient has broad-complex tachycardia features resembling ventricular tachycardia rather than supraventricular tachycardia with a bundle-branch conduction defect.

Which of the following makes Wolff-Parkinson-White the most likely underlying diagnosis?Absence of capture or fusion beat ECG in sinus rhythm reveals right bundle-branch block with left axis deviation QRS duration less than 140 ms P wave preceding wide QRS complex V-lead polarity is discordant

ECG in sinus rhythm showing right bundle branch block and left axis deviation increases the index of suspicion that WPW is the underlying diagnosis.Supraventricular tachycardia with bundle branch block may resemble ventricular tachycardia on the ECG. Eighty percent of all broad complex tachycardias are due to ventricular tachycardia and the proportion is even higher in patients with structural heart disease. Therefore in all cases of doubt, ventricular tachycardia should be diagnosed.The ECG shows a rapid ventricular rhythm with broad (often 0.14 seconds or more) abnormal QRS complexes. AV dissociation may result in visible P waves. Capture beats (intermittent narrow QRS complex owing to normal ventricular activation via the AV node and conducting system) and fusion beats (intermediate between ventricular tachycardia beat and capture beat) are seen. Ventricular tachycardia is more likely than supraventricular tachycardia with bundle branch block when there is:A very broad QRS (> 0.14 seconds) Atrioventricular dissociation A bifid upright QRS with a taller first peak in V1 A deep S wave in V6 A concordant (same polarity) QRS direction in all chest leads (V1 – V6)

Which of the following is a feature of coarctation of the aorta?

If it occurs above the left subclavian artery, blood pressure elevation may be evident only in the left arm It is always associated with a continuous murmur It is often accompanied by a bicuspid aortic valve It presents with the inability to augment cardiac output with exercise Surgical correction usually resolves the hypertension

9

Page 10: Cardiology question

Coarctation of the aorta usually occurs just distal to the origin of the left subclavian artery. If it arises above the left subclavian, blood pressure may be elevated only in the right arm. A continuous murmur is heard only if the obstruction is severe. A diastolic murmur of aortic regurgitation may be heard, as a bicuspid aortic valve may accompany this condition in around 20-30%. Cardiac output response to exercise is not affected unless there is cardiac failure. Hypertension is the major problem and may persist even after complete surgical correction.

A 32-year-old man with Wolff–Parkinson–White syndrome presents with a 2-hour history of palpitations and breathlessness. On examination, his heart rate is 190 beats/min with blood pressure of 100/60 mmHg. ECG shows broad-complex tachycardia.

What would be your first line of treatment? Intravenous amiodarone Intravenous flecainide Intravenous adenosine DC cardioversion Intravenous verapamil

WPW syndrome presents with atrioventricular re-entrant tachycardia caused by an aberrant re-entry circuit via the atrium, atrioventricular (AV) node and ventricle. Orthodromic tachycardia occurs most commonly and is associated with narrow complex SVT, this occurs when conduction goes via the AV node to the ventricles and then back via the accessory pathway. Antidromic tachycardia due to conduction from the atria directly to the ventricle via the accessory pathway is associated with broad complex tachycardia. DC cardioversion is recommended for antidromic tachycardias. Treatment of orthodromic tachycardia aims to block the AV node and cut the re-entry circuit, thereby restoring sinus rhythm. The drug of choice here is adenosine. Flecainide is the drug of choice for atrial fibrillation in WPW syndrome. Cardioversion is only required when drug therapy fails or the patient’s condition is compromised.

A patient with an aortic valve replacement develops right hemiparesis. CT scan shows cerebral infarction. There is no evidence of cerebral haemorrhage. The INR is 2.0.

How would you manage this case? Reverse the anticoagulation with vitamin K Stop warfarin and start intravenous heparin Increase the dose of warfarin Continue warfarin and add intravenous heparin Decrease the dose of warfarin until the INR is 1.5

Use of oral anticoagulation may result in haemorrhage in the infarcted area. As the patient has a mechanical valve, anticoagulation must be continued. The best option therefore would be to stop the warfarin and start intravenous heparin. The reason for this is that heparin can be more easily controlled than oral anticoagulation during the acute period of stroke recovery.

A 45-year-old asthmatic patient presents with palpitations. An ECG shows supraventricular tachycardia, with narrow QRS complexes. Carotid sinus massage is not successful.

What would you do next?Administer intravenous adenosine Administer intravenous verapamil Administer intravenous digoxin Administer intravenous sotolol DC cardioversion

10

Page 11: Cardiology question

Although adenosine is the drug of choice for terminating paroxysmal supraventricular tachycardia, it can cause bronchospasm and is thus contraindicated in patients with asthma, sotolol should be avoided for the same reason. Verapamil would therefore be the drug of choice in this case. However, verapamil should not be used for tachyarrhythmias where the QRS complex is wide. It is also contraindicated in patients with the Wolff–Parkinson–White syndrome.

A 60-year-old man with unstable angina on long-term digoxin was being monitored on the ward with telemetry when the monitor displayed a tachycardia of 180 bpm. The printout showed discrete normal morphology P waves before each QRS complex and there was an acceleration in the rate after initiation of the arrhythmia. The QRS width was 0.12 s.

Which of the following is the most likely arrhythmia? Automatic supraventricular tachyarrhythmias AV nodal re-entrant tachycardia Bypass tract-mediated macroentrant tachycardia Intra-atrial re-entry Ventricular tachycardia

Automatic supraventricular arrhythmias characteristically show a warm-up phenomenon: that is, the rate accelerates after its initiation. Options B, C and D are all types of re-entrant supraventricular arrhythmias. In option B, P waves are usually of abnormal morphology (inverted). In contrast, there are discrete P waves in intra-atrial re-entry but there is no warm-up phenomenon. The normal QRS width rules out a ventricular tachycardia.

A 25-year-old primigravida who is 26 weeks’ pregnant, presents to the casualty department with symptoms of headache, flashing lights and vomiting. Her blood pressure was recorded at 140/100 mmHg and her antenatal diary showed consistent systolic readings of 110–120 mmHg and consistent diastolic readings of less than 80 mmHg. She has a history of mild asthma but was otherwise in good health prior to pregnancy, and there is no family history of note.

Which of the following would be the drug of first choice?ValsartanLabetololMethyldopaNifedipineRamiprilAlthough any of the options from B to D can be used in pre-eclampsia, the drug of reference is is methyldopa, because of its long and reliable track record. However,  labetolol and nifedipine may also be used. In this patient's case, Beta-blockers would be contraindicated due to her history of asthma. ACE inhibitors are contraindicated in pregnancy because they can cause oligohydramnios, renal failure and intrauterine death. Valsartan, (an ARB) is contraindicated. Specialist care is of course required. Magnesium sulphate is used for seizure prevention and treatment, and aspirin may be indicated in some cases. 

A 75-year-old man with isolated systolic hypertension, who also has urinary incontinence, gout and asthma, attends outpatients with a blood pressure reading of 190/86 mmHg.

Which of the following drugs would you initiate for this patient? Amlodipine Atenolol Bendrofluazide Doxazosin

11

Page 12: Cardiology question

Valsartan

Amlodipine, a dihydropyridine calcium-channel blocker, is the drug of choice for the treatment of isolated systolic hypertension in the elderly, especially if thiazides are contraindicated in a patient, as in this man. Doxazosin is contraindicated in patients with urinary incontinence, and, similarly, asthma negates the use of ß-blockers.

A 60-year-old man with NYHA (New York Heart Association) class II heart failure, is taking angiotensin-converting enzyme (ACE) inhibitors and ß-blockers for his heart failure. He is generally well in himself. On direct questioning at his routine outpatient visit, it is noticed that his exercise tolerance has decreased over the last year.

Which of the following drugs should be added to his list of medications? Digoxin Frusemide Isosorbide mononitrate Spironolactone Valsartan

The European Society of Cardiology recommends the addition of spironolactone for improving the survival of patients who are in the transition from well-controlled class II to class III or IV heart failure. Diuretics are only indicated if there is fluid retention. Angiotensin-receptor blockade in addition to ACE (angiotensin-converting enzyme) inhibitors is not recommended at this stage. Digoxin helps to relieve symptoms to some extent, and is more useful if the patient is in atrial fibrillation. Similarly, nitrates and hydralazine help to improve symptoms in patients with class III and IV heart failure.

A 50-year-old man suffers an extensive anterior myocardial infarction but recovers well in hospital. His predischarge ECHO shows him to have an ejection fraction of 35%. He is otherwise asymptomatic.

His medications on discharge should include which of the following? Aspirin, atenolol, ramipril and a statin Aspirin, atenolol, ramipril, frusemide and a statin Aspirin, isosorbide mononitrate, ramipril and a statin Aspirin, nitrate, losartan and a statin Aspirin, atenolol, ramipril, losartan and a statin

There is no evidence to suggest any drug apart from ACE (angiotensin-converting enzyme) inhibitors in asymptomatic left ventricular dysfunction. Following a myocardial infarction, patients with LV dysfunction benefit from taking ß-blockers. There is strong evidence supporting the use of aspirin and a statin post-MI.

A 50-year-old woman who is already on ramipril, frusemide and bisoprolol for heart failure, decompensates and presents to A&E with pulmonary oedema. Her heart rate is 120 bpm and her blood pressure is 100/65 mmHg. She is given oxygen and diamorphine.

Which of the following actions is indicated in her further management? Increase diuretics and maintain the current dose of ß-blocker Increase diuretics, reduce the ß-blocker dose Increase diuretics, increase the ß-blocker dose Increase diuretics, stop ß-blockers and later increase the ß-blocker dose when her lungs are dry Increase diuretics, stop ß-blockers and restart ß-blockers when her lungs are dry

12

Page 13: Cardiology question

Although ß-blockers are not started for the first time in a patient with pulmonary oedema, in a case like this, when the patient is already on ß-blockers, it is wise to carry on with the same dose.

Which of the following patients would be best served by a permanent pacemaker?

40-year-old man with third-degree AV block and a maximum documented period of asystole of 1.5 s 40-year-old man with type II second-degree AV block and an escape rate of 30 bpm when awake and asymptomatic 40-year-old man with Lyme disease having symptomatic complete AV block 40-year-old man with chronic asymptomatic trifascicular block and first-degree AV block 40-year-old man 3 days after suffering an acute anterior MI and having a persistent first-degree AV block and old right bundle-branch block

Third-degree and advanced second-degree atrioventricular block associated with any of the following conditions definitely needs a permanent pacemaker: • symptomatic bradycardia• documented periods of asystole of 3 s or more• any escape rate less than 40 bpm in awake, asymptomatic patientsIn Lyme disease, the AV block is usually temporary and so does not need permanent pacing. Chronic asymptomatic bi- and trifascicular block needs pacing if associated with type II second-degree or third-degree AV block, but not otherwise. Following an acute myocardial infarct, pacing is generally indicated for a second- and third-degree block only at or below the AV node level.

A 50-year-old man has effort-related angina. His total cholesterol is 5.5 mmol/litre. He has no other cardiac risk factors and no other relevant medical history.

Which of the following is the most appropriate initial treatment?ß-blocker and statin Calcium-channel blocker and nitrate  Diltiazem and statin  Nitrate, ß-blocker and calcium-channel blocker  Nitrate and statin Although experts may use any of the combinations, the choice of first-line treatment for the prophylaxis of effort-related angina in someone without contraindications is a ß-blocker. ß-Blockers also remain the choice in someone who has suffered a previous myocardial infarction. Calcium-channel blockers – especially diltiazem and verapamil can be used, although they should be avoided in the presence of significant left ventricular dysfunction. Nitrates are of good value, but they come after ß-blockers. Care should be taken before combining a ß-blocker and a calcium-channel blocker, especially if there is evidence of conduction disturbance or LV dysfunction. The Scandinavian Simvastatin Survival Study showed that a statin given to patients with angina pectoris and a cholesterol level of 5.5–8.0 mmol/l (212–308 mg/dl) significantly reduced the risk of MI. Aspirin would also of course be recommended for secondary prevention of ischaemic events and nitrates could be used for symptom relief.

An 80-year-old man has isolated systolic hypertension. He also suffers from angina, gout and peripheral vascular disease.

Which of the following antihypertensives is best suited for him initially? Bendrofluazide Frusemide Atenolol Nifedipine Ramipril

13

Page 14: Cardiology question

Dihydropyridines are especially suited for elderly patients with isolated systolic hypertension, angina and peripheral vascular disease. Thiazide diuretics are contraindicated in gout; ß-blockers are relatively contraindicated in peripheral vascular disease.NB: In the given setting, ACE inhibitors would not be a first-choice agent, but if the question was worded in such a way that the patient had congestive cardiac failure or type 1 diabetic nephropathy, then the choice would have been ramipril.

An article in a leading medical journal reads: ‘an insertion of 5 nucleotides in the gene was identified as the cause of hypertrophic cardiomyopathy in this family’.

Which of the following type of mutation is the author referring to? Frame-shift mutation Repeat mutation Missense mutation Nonsense mutation Point mutation

Because the group of nucleotides inserted is not a multiple of three, the mutation changes the frame in which translation occurs and hence the name ‘frame-shift mutation’.A Missense mutation is an alteration in a nucleotide sequence that converts a codon for one amino acid into a codon for a second amino acid. An insertion mutation arises by the insertion of one or more nucleotides into a DNA sequence. A nonsense mutation is an alteration in nucleotide sequences that changes a triplet coding for an amino acid into a termination codon. A point mutation results from a single nucleotide change in a DNA molecule.Another type of mutation relevant for MRCP Part 1 is a read-through mutation, which changes a termination codon into a codon specifying an amino acid and hence results in read-through of the termination codon.

Which of the following conditions is most likely to produce a wide, relatively fixed split of S2?

Congestive cardiac failure Left bundle-branch block Moderate ventricular septal defect Right bundle-branch block and heart failure Wolff–Parkinson–White syndrome

RBBB makes the split wide and the heart failure fixes the split. (Heart failure does not permit much of a change in ventricular volume with respiration, because breathing with congested lungs is shallow.) Right ventricular failure (RVF) secondary to pulmonary hypertension (PHT) is another condition where you can get a wide fixed split, the wideness of the split being due to a prolonged isovolumetric contraction time in the failing right ventricle. In cases of moderate ventral septal defects, the widening is due to delayed P2, but it is not fixed. In WPW and other causes of electrical delay of LV conduction, including LBBB, there is a narrowly split S2.

What does a prominent left precordium in a 16-year-old young man with an ejection murmur in the second left intercostal space indicate?

ASD with aortic regurgitation ASD with aortic stenosis ASD with mitral stenosis ASD with pulmonary hypertension Uncomplicated ASD

14

Page 15: Cardiology question

A prominent left precordium suggests that the right ventricle was dilated during childhood, and also that it was working against a high pressure. Ostium secondum atrial septal defect (ASD) in combination with rheumatic mitral stenosis (Lutembacher’s syndrome) can cause the same picture in advanced cases when there is pulmonary hypertension, but option D is a better choice than C.

Which of the following pharmacological agents is most likely to benefit a patient with angina due to cardiac syndrome X?

Aspirin Bisoprolol Diazepam Diltiazem Isosorbide mononitrate

Nitrates are often effective in patients with syndrome X. Cardiac syndrome X consists of angina-like chest pain during exertion, characteristic ECG changes during exercise testing, normal coronary arteries on cardiac catheterisation and no inducible coronary artery spasm during catheterisation. It should not be confused with the metabolic syndrome X, which comprises central obesity, glucose intolerance, dyslipidaemia, and high blood pressure. The dyslipidaemia in this case is primarily high triglycerides and low HDL cholesterol. People with metabolic syndrome are at increased risk of coronary artery disease.

A patient has tuboeruptive xanthomas, distributed subcutaneously and mainly on the extensor surface of extremities.

What is the probable diagnosis? Type I hyperlipoproteinaemia Type II hyperlipoproteinaemia Type III hyperlipoproteinaemia Type IV hyperlipoproteinaemia Type V hyperlipoproteinaemia

Tuboeruptive xanthomas occur in type III hyperlipoproteinaemia. Eruptive xanthomas are associated with hyperchylomicronaemia (type I and type V hyperlipoproteinaemia). Xanthoma tendinosum, which are nodular swellings of tendons, usually occur in type II hyperlipoproteinaemia.

To establish the aetiology of pulmonary hypertension, a cardiac catheter study was performed. The wedge pressure was normal and the mean mitral valve diastolic pressure gradient was > 3 mmHg at rest, both of which increased with exercise.

From this data, what is the probable diagnosis? Congenital heart disease Left ventricular diastolic dysfunction Major pulmonary artery occlusion Mitral regurgitation Mitral stenosis

The haemodynamic pattern is typical of mitral stenosis and may also be seen in cor triatriatum, in which there is a left atrial membrane. A large systolic pressure wave in wedge tracing can be seen in patients with mitral regurgitation. In addition, regurgitation of contrast from a left ventricular angiogram to the left atrium can be seen. The left ventricular end-diastolic pressure (LVEDP) is > 15 mmHg in those with LV diastolic dysfunction. There will be a focal pressure gradient in a lobar or larger pulmonary artery, intravascular filling defect or narrowing in patients with major pulmonary artery occlusion by clot or

15

Page 16: Cardiology question

tumour. In congenital heart disease, there will be a step up in oxygen saturation in the right heart and a step down in the left heart.

A 17-year-old young man presents with palpitations. His physical examination is normal except for a systolic murmur in the second left intercostal space and prominent precordial motion with a late systolic impulse.

Which of the following conditions is he likely to have?Aortic stenosis Atrial septal defect Hypertrophic cardiomyopathy Mitral valve prolapse Mixed aortic valve disease

A mid to late systolic impulse in the precordial motion (triple ripple) is seen in patients with HOCM. A hyperdynamic impulse is seen in mild to moderate AR. Conditions A, B, D and E can produce the murmur but not this characteristic finding.

A 25-year-old woman is seen in outpatients and is found to have loud first heart sound, an early diastole sound followed by a mid-diastolic murmur.

What is the likely diagnosis? Mitral stenosis with a fourth heart sound Mitral stenosis with atrial fibrillation Mitral stenosis with mobile leaflets Mitral stenosis with pulmonary hypertension Mitral valve prolapse

From the information given, the diagnosis is mitral stenosis with mobile leaflets. There is no information regarding atrial fibrillation or pulmonary hypertension. There is a loud first heart sound and when an opening snap is heard this indicates that the mitral valve is mobile. The snap occurs when the superior systolic bowing of the anterior mitral valve leaflet is rapidly reversed towards the left ventricle in early diastole, due to the high left atrial pressure. S4 is classically late diastolic or presystolic. It occurs when augmented atrial contraction causes presystolic ventricular distension so that the ventricle then contracts with greater force. Atrial myxoma can produce an early diastolic sound (tumour plop), which is due to the abrupt diastolic seating of the tumour within the right or left atrioventricular orifice.

Which of the following microanatomical structures within the heart interacts with conventional calcium-channel blockers?

L type Calcium-channels Calcium-channel T type T tubules Titin Tropomyosin

The T tubules are a tubular network formed by the invagination of the sarcolemma of the myocyte. Sarcolemmal calcium channels are located on the T tubules; there are two main types of channels – T and L types. The T (transient) channels do not interact with conventional calcium-channel blockers. Calcium-channel blockers interact with the L-type calcium channels. Titin tethers the myosin molecule to the Z line, and its elasticity explains the stress–strain elastic relation of striated muscle. It is the largest protein molecule yet described. The thin actin filaments intertwine and are carried on a heavier tropomyosin molecule that functions as a backbone. At regular intervals along this structure is a group of three

16

Page 17: Cardiology question

regulatory proteins called the ‘troponin complex’, which is composed of troponin C, troponin I and troponin M.

Which of the following best describes the mechanism of action of flecainide as an antiarrhythmic agent? Slows the upstroke of the action potential Increases the action-potential duration Has a direct membrane effect Increases vagal tone Affects SA and AV nodes

Flecainide, a class Ic agent slows the upstroke of the action potential and is its main mechanism of action. It has minimal effects on action-potential duration. In other words, it causes a marked decrease in conductivity, with little effect on refractoriness. The antiarrhythmic group that mainly affects sinoatrial and atrioventricular nodes, and thus has a direct membrane effect, is the calcium-channel blockers. Class V agents (digitalis agents) affect SA and AV nodes by increasing vagal tone.

Which of the following antiarrhythmics have the highest risk of producing torsades de pointes?

FlecainideLidocainePhenytoinPropafenoneSotalol

Among these agents sotalol, which is a class III agent, has the highest risk of producing torsades, which is a polymorphic, pause-dependent ventricular tachycardia causing syncope and sudden cardiac death. Lidocaine and phenytoin (which are class Ib agents) and flecainide and propafenone (class Ic) have an almost zero risk of producing TdP.

Urinary hesitancy as a sign of drug-induced toxicity is characteristic of which of the following antiarrhythmics?

AmiodaroneSotalolDisopyramideFlecainideVerapamil

Amiodarone causes hepatic effects, peripheral neuropathy, proximal myopathy, thyroid dysfunction, skin discoloration and pneumonitis, among others. Sotalol (ß-blockers) and flecainide have negative inotropy and CNS effects. Verapamil causes bradycardia.

A 25-year-old medical student noticed that he had a murmur when he tested his new stethoscope. On assessment in the cardiology clinic, he was found to have a harsh systolic murmur over his precordium, which did not change with inspiration. ECG showed features of biventricular hypertrophy.

What is the most likely diagnosis? Aortic stenosis Hypertrophic cardiomyopathy Mitral regurgitation Tricuspid regurgitation

17

Page 18: Cardiology question

Ventricular septal defect

Right-sided murmurs increase with inspiration (eg tricuspid regurgitation, TR), whereas left-sided murmurs show no change. The clue to diagnosis is in the ECG finding. Aortic stenosis and mitral regurgitation produce left ventricular hypertrophy, TR produces right ventricular hypertrophy and a ventricular septal defect produces biventricular hypertrophy.

A 25-year-old man presents to the emergency department wth a 1-week history of fever and myalgia. He had travelled to Chile 8 weeks ago. On examination there are no positive findings, although the patient recollects that his right eyelid was swollen for a few weeks after he left Chile. ECG reveals non-specific, T-wave changes in all leads.

What is the most likely diagnosis? EchinococcosisFalciparum malaria SchistosomiasisToxoplasmosisTrypanosomiasis

Trypanosoma cruzi. causes American trypanosomiasis or Chagas disease and is quite common in South America. The vectors are reduvid bugs. The trypanosomes are transmitted by scratching infected faeces of the bug into skin abrasions caused by the bug during blood sucking. In acute trypanosomiasis, the patient presents with fever, myalgia, hepatosplenomegaly and myocarditis. Unilateral periorbital oedema and swelling of the eyelid can result from a bug bite around the eyes. This is called Romana’s sign. The other conditions listed can cause myocarditis, but the best choice is trypanosomiasis.

A 30-year-old woman presents with pleuritic chest pain and haemoptysis. Her blood pressure is stable at 130/80 mmHg. A ventilation/perfusion scan shows minor mismatch at the lung bases. There is no evidence of RV dysfunction, clinically and on echocardiography. In addition to oxygen, which of the following is the appropriate management for this patient?

Heparin and consideration for surgeryHeparin and paracetamolHeparin plus mechanical interventionHeparin plus thrombolytic therapySupportive

This patient has potentially had up to two small to moderate pulmonary embolisms, probably associated with pulmonary infarction. The management in this case would be heparin and simple analgesics to control her chest pain. If there were signs of a large/massive PE (hypotension, right ventricular dysfunction), the ideal management would include thrombolytic therapy or mechanical intervention.

A 50-year-old man underwent coronary artery bypass grafting 2 days ago. A routine liver function test result now shows that both the direct and indirect bilirubin are elevated. All the other liver function tests are normal.

Which of the following is the most likely cause? ‘Shock liver’ syndrome Anaesthetic-inducedHaemolysis on bypass Narcotic-inducedRight heart failure

18

Page 19: Cardiology question

Isolated elevation of direct and indirect bilirubin, indicates haemolysis on the cardiopulmonary bypass and can be confirmed by increased plasma free-haemoglobin levels. There is no specific treatment. Markedly raised enzyme levels are seen in patients with the ‘shock liver’ syndrome, and the treatment is aimed at maximising cardiac output and oxygenation. Right heart failure is another cause of hyperbilirubinaemia in the immediate post-bypass period, and, in this case, the direct bilirubin and alkaline phosphatase are increased without enzyme elevation. Treatment is as for right heart failure.

In an asymptomatic patient, a permanent pacemaker is indicated in which of the following conditions?

First-degree block at the AV node First-degree block in the distal conduction system with an HV interval < 100 ms Second-degree block at the AV node Second-degree block at the distal conduction system

Right bundle branch block

In an asymptomatic patient, a permanent pacemaker (PPM) is indicated in second- and third-degree heart block at the distal conduction system. If the third-degree block at the AV node had been associated with symptoms, it would have been an indication for PPM. Also, a PPM is indicated for cases of first-degree AV block in the distal conduction system with an HV (Bundle of His to ventricular depolarisation) interval of greater than 100 ms associated with symptoms.

A 70-year-old man, previously fit and well, is referred to out-patients with exertional chest pain that comes on at around 0.8 km (0.5 mile) on the flat. Examination reveals him to be in sinus rhythm at 80 bpm, blood pressure 100/70 mmHg, a diminished carotid upstroke and a loud ejection systolic murmur over the aortic area. Echo confirms left ventricular hypertrophy, preserved systolic function with peak aortic valve gradients of 80 mmHg. Subsequent coronary angiography reveals significant disease in the left anterior descending artery.

What is the optimal treatment strategy? Angioplasty and stenting to the left anterior descending artery Aortic valve replacement and bypass graft Aspirin and review in 3 months’ time with a repeat echo Aspirin and a b-blocker, and review in 3 months’ time with a repeat echo Percutaneous aortic valvotomy

This patient has severe (aortic valve gradient > 70 mmHg), symptomatic aortic stenosis and as such valve replacement is indicated, unless precluded by co-morbidity. Coronary angiography is performed to assess the need for concomitant coronary artery bypass grafting. With acquired aortic stenosis, patients most commonly present in their sixth decade with symptoms of angina, syncope or heart failure. The development of symptoms is associated with a poor outcome if left untreated. The average life expectancy from the onset of symptoms to death is 2 years in patients with heart failure, 3 years in those with syncope and 5 years in those with angina.

A 52-year-old woman, with a prior history of rheumatic fever, presents with shortness of breath on strenuous exertion while working as a landscape gardener. She is in permanent atrial fibrillation and is on long-term warfarin and digoxin (125 mg once daily). Clinical examination reveals her to be in atrial fibrillation at a rate of around 150 bpm. Echo demonstrates preserved left ventricular function, a heavily calcified mitral valve with moderate mitral stenosis (mitral valve area 1.5 cm2) and moderate mitral regurgitation. Her left atrium is dilated.

What is the most appropriate initial treatment option? AmiodaroneAtenololDC shockMitral valve replacement

19

Page 20: Cardiology question

Percutaneous mitral valvotomy

This woman has moderate, mixed mitral valve disease and therefore surgery is not currently indicated. Even if the mitral stenosis were to be severe the presence of heavy calcification of the valve and concomitant mitral regurgitation would preclude percutaneous valvotomy. She is in permanent atrial fibrillation and as such, by definition, sinus rhythm cannot be restored (as opposed to persistent or paroxysmal). Better rate control is required. Digoxin on its own may not control catecholamine-driven tachycardia (eg during exertion). Amiodarone, while effective, would not be the first choice for this young patient working outdoors because of its side-effect profile.

A 72-year-old Caucasian man is referred to out-patients with a 6-month history of progressive exertional dyspnoea. His ankles swell as the day progresses. There is no associated chest discomfort. He is an ex-smoker of 3 years and drinks 12 pints of beer per week. He has not seen his GP in the previous 15 years. The only past history is that of mild asthma as a child. His father died of a myocardial infarct aged 65 years. Blood pressure is 150/86 mmHg. Results of investigations are as follows: renal function, normal; cholesterol, 6.8 mmol/l; ECG, sinus rhythm LBBB; echo, dilated and impaired left ventricular function with ejection fraction of 30%, mild to moderate mitral regurgitation, no LVH.

What is the most likely underlying aetiology? AlcoholCoronary artery disease HypertensionValvular heart disease Viral myocarditis

It is important to remember that chronic heart failure is not an absolute diagnosis; there is a cause to be found. In the UK the commonest aetiology is underlying ischaemic heart disease (around 50–66% of all cases). This patient has several risk factors for ischaemic heart disease, which adds weight to the likelihood of this being the diagnosis. It is of course important to exclude other potentially reversible causes such as thyroid disease, ethanol, nutritional deficiencies, primary valvular pathology and uncontrolled prolonged tachycardia. The mitral regurgitation in this patient is most likely to be secondary to left ventricular dilatation and subsequent annular dilatation of the mitral valve ring. This will contribute to the development of pulmonary hypertension.

A 50-year-old man with long-standing hypertension presents acutely with severe chest pain radiating through to his back. He looks unwell, with a resting tachycardia (110 bpm) and blood pressure of 150/96 mmHg. There are no murmurs and neurological examination is normal. An urgent CT scan of his chest confirms type-A aortic dissection. The local cardiothoracic centre is contacted and urgent transfer arranged. He has received appropriate opiate analgesia.

What additional drug treatment should be instigated as part of his immediate treatment plan? Intravenous GTN Intravenous labetalol Intravenous nitroprusside Oral amlodipine Oral enalapril

Type-A dissection involves the ascending aorta, and treatment with urgent surgical intervention is recommended unless severe co-morbidity would preclude the patient from surgery. Immediate management includes the liberal use of opiates for complete pain relief, thereby decreasing the sympathetic drive. Meticulous blood pressure control is vital while awaiting surgery/transfer, in an attempt to reduce the chances of extension or rupture. Short-acting intravenous b-blockers (eg labetalol) are the first choice drugs as they reduce both blood pressure and force of ejection. A short half-life is important since haemodynamics can change rapidly. If b-blockers are contraindicated then sodium nitroprusside or calcium-channel blockers may be appropriate alternatives.

20

Page 21: Cardiology question

Cardiac catheterisation is performed on a 25-year-old man with a systolic murmur but no symptoms. ECG and chest X-ray are normal. The findings are as follows (pressures mmHg): aorta, 125/70; left ventricle, 120/12; right atrium, mean 8; right ventricle, 40/8; pulmonary artery, 44/14; pulmonary capillary wedge, mean 13. Saturations (%): aorta, 97; superior vena cava, 68; right atrium, 70; right ventricle, 82; pulmonary artery, 85.

What is the most likely cardiac diagnosis? ASD – primum ASD – secundum Mitral stenosis Primary pulmonary hypertension Ventricular septal defect

The catheter data demonstrate a step up in saturations from the right atrium to the right ventricle, in keeping with a left to right shunt at the level of the ventricle. There is a mild elevation in pulmonary artery pressure in keeping with the shunt. VSD is the commonest form of congenital heart disease. In adults a small defect may present as an asymptomatic murmur; or in the extreme as Eisenmenger’s syndrome, where reversal of a left to right shunt has occurred as a consequence of advanced pulmonary hypertension. VSD is associated with an increased risk of endocarditis although recent NICE guidelines do not say that routine antibiotic prophylaxis is recommended. Management depends upon the actual size of the shunt.

A 70-year-old woman is admitted to hospital with a swollen left leg 4 weeks after undergoing an elective total hip replacement. An above-knee DVT is diagnosed by ultrasound. She is in sinus rhythm at 60 bpm and her blood pressure is 160/80 mmHg. She is commenced on the appropriate dose of low molecular weight heparin and warfarin loading. The following day she becomes acutely short of breath. Examination reveals a resting tachycardia (110 bpm) with blood pressure of 100/60 mmHg. Her JVP is elevated at 7 cm above the sternal notch. Arterial blood gas measurement reveals her to be hypoxaemic with a pa(O2 ) of 7 mmHg.

What would be the first-line therapy after administering high-flow oxygen? AspirinIntravenous heparin Surgical embolectomy Thrombolysis with reteplase Vena caval filter

This patient has clinical features of a massive pulmonary embolus. This results from significant obstruction of the pulmonary arteries causing haemodynamic compromise – namely shock or systemic hypotension (systolic blood pressure < 90 mmHg or a drop of > 40 mmHg for > 15 minutes). The initial treatment of choice is thrombolysis using a recognised protocol. Whilst she is only 4 weeks out from her hip replacement, the benefits fo thrombolysis would outweigh the risks in this case. Inotropic support and the judicious use of fluids may also be required in the interim. Subsequent intravenous unfractionated heparin should then be commenced.

A 70-year-old man is referred by his GP for advice regarding optimisation of secondary prevention. He has a history of angina, with excellent control of symptoms on a combination of aspirin, dipyridamole MR, atenolol 50 mg od, simvastatin 40 mg od and isosorbide mononitrate 20 mg bd. His pulse rate is 70 bpm and blood pressure is 144/86 mmHg. The only other relevant past history includes an ischaemic stroke 2 years ago from which he made a complete recovery.

What additional therapy would you consider adding? BendroflumethiazideDiltiazemDoxazosin

21

Page 22: Cardiology question

NicorandilPerindopril

Most clinicians would now recommend the addition of ACE inhibitors for patients with vascular disease, irrespective of left ventricular function. This is based on evidence from large trials, such as PROGRESS (perindopril) and HOPE (ramipril). Favourable outcomes were found to be independent of a blood pressure effect. As such, the benefit of ACE inhibition seems to be not purely related to a reduction in blood pressure; beneficial local vascular and myocardial effects are also seen. Blood pressure is not yet optimised in this patient and further antihypertensive therapy is warranted. The addition of an ACE inhibitor should bring this to the desired level (< 140/85 mmHg).

An 81-year-old woman is referred to cardiology out-patients with a history of dizzy episodes and one episode of syncope. She is known to have long-standing atrial fibrillation. A 24-hour tape confirms atrial fibrillation, with rates varying from 30 to 140 bpm. There are several daytime pauses of over 3 seconds. She is listed for a permanent pacemaker.

Which of the following would be the most appropriate device? DDDDDIVOOVVIVDD

Pacemaker types are defined by an international code, which uses three or more letters in a set sequence. The first letter is related to the chamber that is to be paced: A = atrium, V = ventricle, D = both. The second letter refers to the chamber that is sensed (A, V or D). The third letter refers to the response to a sensed beat by the pacemaker: I = inhibits, T = trigger or D = both (ie either inhibits or triggers). VOO is a fixed output setting (eg pacing at 60 beats per minute irrespective of intrinsic activity); the ‘O’ as second and third symbol implies that the chamber is not sensed and therefore there is no response to a sensed beat. A fourth letter refers to whether or not the pacemaker has rate-adaptive properties (R). This woman has atrial fibrillation and as such there is no need for an atrial lead. VVI means there is one lead in the ventricle (pacing and sensing the ventricle). If the pacemaker senses an intrinsic QRS complex then pacing is inhibited (I). This means the pacemaker will only deliver a stimulus when there is a significant pause.

A 68-year-old man is admitted with syncope. He is known to have ischaemic cardiomyopathy. His medications include: aspirin 75 mg od, frusemide 80 mg bd and lisinopril 10 mg od. An initial ECG shows sinus bradycardia (50 bpm) and RBBB. Results of blood tests are as follows: sodium, 134 mmol/l; potassium, 3.5 mmol/l; creatinine 124 mmol/l. He has recurrent syncopal episodes on the CCU, where monitoring shows episodes of non-sustained torsades de pointes (polymorphic VT).

Which of the following would be your initial line of treatment? DC cardioversion Intravenous amiodarone Intravenous magnesium Oral metoprolol Temporary pacing

Torsades de pointes (polymorphic VT with QRS complexes of different amplitude twisting around isoelectric line) occurs in patients with a prolonged QT interval. Any cause of QT prolongation can predispose to the arrhythmia.  These include: congenital (the Jervell–Lange-Neilsen or Romano–Ward syndromes); metabolic (hypo-calcaemia, -magnesaemia or –kalaemia); drugs (eg amiodarone, tricyclic antidepressants, phenothiazines); ischaemic heart disease; mitral valve prolapse. The arrhythmia often occurs in the context of bradycardia. The key here is that amiodarone may exacerbate the situation. Intravenous magnesium (even if the serum magnesium concentration is normal) is the first-line therapy.

22

Page 23: Cardiology question

Temporary pacing at higher rates with or without b-blockers is the next line of therapy. DC shock would not be helpful since episodes are non-sustained.

A 78-year-old woman presents to A&E with three episodes of syncope in the last 24 hours. There is no history of chest pain. She is taking frusemide 80 mg od and ramipril 10 mg od for known hypertension. She is conscious with a blood pressure of 100/40 mmHg. Potassium is 5.3 mmol/l. Her ECG shows complete heart block with rate of 40 bpm. QRS duration is 150 ms with a right bundle-branch block configuration.

What is the optimum initial management? DobutamineIsoprenalineIntravenous calcium chlorideTemporary transvenous pacingWithhold medication and observe This woman has complete heart block with an unstable escape rhythm. The latter is exemplified by the fact that she has already had three syncopal episodes. Her QRS duration is prolonged (normal up to 120 ms), and this is generally more unstable than an escape rhythm of normal duration (ie < 120 ms) since this originates from around the His bundle. Her blood pressure is low, particularly with a background of hypertension. In addition, it is important to remember that cardiac output will be influenced by heart rate. In the elderly, cerebral vascular dysregulation may compound the effect thereby contributing to cerebral hypoperfusion. Ideally she should receive a transvenous temporary pacemaker. If further acute problems occur while waiting for a transvenous temporary pacemaker (eg awaiting transfer to a room with fluoroscopy), then external pacing can be instituted in the short term with appropriate sedation.

A 50-year-old man presents with a 1-hour history of severe central chest pain. There is no significant past medical history. He is haemodynamically stable with pulse rate of 90 bpm and blood pressure of 120/70 mm Hg. ECG shows 5 mm of ST-segment elevation in the anterior leads (V2–V4). He received aspirin 300 mg in the ambulance and 5 mg diamorphine.

What would be the next line of treatment? ClopidogrelEnoxaparinGIIb/IIa blocker StreptokinaseTissue plasminogen activator

This relatively young man has presented early with acute anterior myocardial infarction. The key therapeutic aim is early reperfusion in an attempt to save myocardium. In centres with rapid access to primary angioplasty this would be the optimum strategy. However, in the UK t his is a rarity and far from the norm. All patients with suspected MI should receive aspirin. In youngish* patients presenting early (within 4 hours) with acute anterior MI, thrombolysis with accelerated tissue plasminogen activator (tPA) should be considered since it is associated with a small benefit in the mortality rate over streptokinase. The downside is the cost; tPA is several times more expensive than streptokinase. *The evidence (GUSTO trial) is for patients less than 55 years of age, male, and within 4 hours but most would agree for patients less than 60 years of age.

A 72-year-old Caucasian woman is referred to out-patients for advice regarding her hypertension management. She has been on treatment in the form of perindopril 4 mg od for the past 3 years. However, on repeated measurements, her readings have been > 160 mmHg systolic, with diastolic readings being in the order of 80–85 mmHg. Renal function is normal as is urine dipstick testing. There is no evidence of left ventricular hypertrophy on ECG. She is obese with a BMI of 33.

What would you consider adding as your next drug? Atenolol

23

Page 24: Cardiology question

BendrofluazideDoxazosinAmlodipineSpironolactone

This woman has hypertension resistant to a single agent. It is increasingly recognised that more than one agent is required to adequately control blood pressure. Whilst guidelines suggest either a thiazide or calcium channel antagonist could be added next, in view of the fact she is obese, a calcium channel antagonist may be the better choice. Meta-analyses have shown that both thiazides and beta blockers are associated with an increased risk of the development of Type 2 diabetes in at risk patients. Indeed, the ASCOT study did suggest that the combination of ACE inhibitor and calcium antagonist was associated with the development of less type 2 diabetes than a beta blocker/thiazide alternative.

A 53-year-old bus driver presents with a history of chest pain at rest. Initial ECG shows minor ST-segment depression in the lateral leads. Cardiac enzymes, including troponin, are normal. He has known angina, with angiography 3 years previously demonstrating minor right coronary artery disease. He is commenced on aspirin, a b-blocker and a statin. His symptoms settle over 24 hours, 12 hr troponin is normal, and following mobilisation he is discharged home. He needs to know how this episode might affect his future employment.

Assuming his condition remains stable with no further symptoms, what would be hte next stage of investigation?AngiographyEchocardiogramExercise testingMyocardial perfusion imagingReview in outpatients after 6 weeks

This man holds a group 2 licence, and hence he should be advised to inform the DVLA of his recent presentation with unstable angina and stop work until re-licensing can be performed. Up-to-date information can be obtained from the DVLA. For group 2 licence holders, all acute coronary syndromes are considered relevant and this disqualifies the individual from driving for at least 6 weeks. Re-licensing may be permitted if a suitable exercise test is achieved (need to complete three stages of the Bruce protocol or equivalent safely, without antianginal medication for 48 hours and without significant symptoms, ECG or haemodynamic abnormalities) and there are no other disqualifying conditions. Although angiography is not required for re-licensing, if it is performed then specific guidelines are available on the DVLA website. In this case he would be exercised at the 6 week stage.

Which one of the following statements is MOST accurate regarding coarctation of the aorta?

The coarctation is proximal to the left subclavian artery origin if the right arm blood pressure is significantly higher than in the left arm Continuous murmur over the thoracic spine usually originates from extensive collaterals Rib notching on plain chest X-ray can be identified as early as three months after birth Atrial septal defect (ASD) is the commonest associated congenital abnormality The risk for bacterial endocarditis means that antibiotic prophylaxis is required prior to all dental procedures

The commonest site of discrete obstruction of the aortic lumen is just distal to the origin of the left subclavian artery. The systolic arterial pressure in the arms exceeds that in the leg. However, if the systolic arterial pressure in the right arm is higher than that of the left arm by more than 30 mmHg, the left subclavian is involved in the coarctation (i.e. coarctation is proximal to the origin of the subclavian as in this case). A continuous murmur over the thoracic spine usually originates from small, tight coarctation (< 2 mm). Other cardiac malformations are frequent, the commonest being a bicuspid aortic valve. Notching of the inferior border of the ribs from collateral vessels is common and usually manifest in adults and older children. Patients with unrepaired coarctation are at risk of bacterial endocarditis

24

Page 25: Cardiology question

and should be advised about this possibility. Antibiotic prophylaxis is no longer advised given the poor evidence that this is effective in reducing the risks of infection following dental or other procedures.

Which one of the following is characteristic of atrial myxoma?

Usually originates in the right atrium Fragments of tumour easily break off and grow in its peripheral sites Echocardiogram is diagnostic in most cases The clinical signs can mimic severe mitral regurgitation Recurrence is frequent even after successful surgical removal of the tumour Atrial myxoma is a benign tumour of the heart. Approximately 75% originate in the left atrium. The clinical features are characterised by a triad of embolism, intracardiac obstruction and constitutional symptoms. The clinical signs can mimic mitral stenosis and the murmur may vary with body position. Fragments of tumour easily break off but do not grow in its peripheral sites. After complete and careful removal of the tumour recurrence is very rare.

A 30-year-old woman presents with a three month history of chest pain. On auscultation, there is a midsystolic click and a late systolic murmur. Her electrocardiogram shows T-wave inversions in leads II, III, and aVF.

Which of the following statements concerning her condition is true? The woman’s chest pain could be due to associated coronary artery disease The click and murmur is likely to occur earlier in systole when the patient stands An exercise stress test would most likely be positive Asymmetrical hypertrophy of the interventricular septum is revealed on echocardiography Prophylactic measures to prevent subacute bacterial endocarditis are not warranted

The systolic click-murmur syndrome is associated with mitral valve prolapse. It occurs in approximately 4% of the normal asymptomatic population. It can place excessive stress on the papillary muscles and lead to ischaemia and chest pain. Although often associated with inferior Twave changes, the systolic click-murmur syndrome only occasionally results in an ischaemic response to exercise. On standing or during the Valsalva manoeuvre, as ventricular volume gets smaller, the click and murmur move earlier in systole. Echocardiography reveals mid-systolic prolapse of the posterior mitral leaflet or, on occasion, both mitral leaflets into the left atrium. Asymmetrical hypertrophy of the interventricular septum is a feature of hypertrophic obstructive cardiomyopathy (HOCM). Infective endocarditis prophylaxis is necessary for those patients with a murmur; an isolated mid-systolic click does not merit them.

Normal pregnancy is associated with which one of the following haemodynamic changes?

A 20% reduction in blood volume and cardiac outputA 10 mmHg drop in diastolic blood pressure during the second trimesterBradycardia with a radial pulse rate between 45 and 55 beats per minuteGrade 2/6 diastolic murmur at the mitral areaPulsus alternans

Despite an expansion of the plasma volume and cardiac output of 50%, mean and diastolic blood pressures fall by approximately 15% owing to a reduction in peripheral vascular resistance. There is little change in systolic blood pressure, but diastolic pressure is reduced (5-10 mmHg) from about 12-16 weeks. Diastolic pressure usually increases thereafter to pre-pregnancy levels by about 36 weeks. Tachycardia rather than bradycardia is a recognised physiological change during pregnancy. It is a consequence of reduced peripheral vascular resistance and fall in blood pressure levels. The heart may be slightly enlarged and may be displaced outward because of the high diaphragm. A pulmonary systolic murmur from a high blood flow is common and there may be a physiological third heart sound. Diastolic murmurs

25

Page 26: Cardiology question

are generally pathological and at the mitral area may signify mitral stenosis. The presence of pulsus alternans usually signifies advanced heart failure.

Which one of the following features is MORE common in constrictive pericarditis than in cardiac tamponade?

Pulsus paradoxus Kussmaul’s sign Prominent x trough 4-chamber diastolic equilibrium Right-sided heart failure

An inspiratory increase in venous pressure (Kussmaul’s sign) and a steep y descent in the jugular pulse are features of constrictive pericarditis. Pericardial knock in early diastole is often seen in constrictive pericarditis. Both conditions cause failure of either side of the heart and the diastolic pressure in all cardiac chambers are equal. A paradoxical pulse and prominent x trough in the jugular pulse are more common in tamponade than in constrictive pericarditis.

A 45-year-old woman is being investigated for heart disease. It is found that the pressure–volume curve of the left ventricle is shifted to the left.

What is the most likely diagnosis in this case? Aortic regurgitation Mitral stenosis Aortic stenosis Mitral regurgitation Tricuspid stenosis

The pressure–volume curve denotes the pumping mechanics of the heart chambers. Most studies refer to the left ventricle, as this is the main chamber. In aortic stenosis, there is pressure overload leading to concentric hypertrophy of the left ventricle. This causes increased contractility and decreased compliance of the chamber. More pressure is thus exerted to eject the same volume of blood. The pressure–volume curve therefore shifts to the left.In aortic and mitral regurgitation, volume overload occurs leading to a dilated left ventricle. This causes decreased contractility and increased compliance and shifts the pressure–volume curve to the right. In mitral and tricuspid stenosis, there is pressure overload in the left atrium and not the left ventricle. There would thus be no change in the pressure–volume curve of the left ventricle.

The pressure–volume curve in a patient with heart failure is shifted to the right.

What is the most important feature in cardiovascular dynamics responsible for this right shift? Increased contractility of the chamber Increased sympathetic activity Concentric hypertrophy of the chamber Increased compliance of the chamber Pressure overload in the chamber

Shifting of the pressure–volume curve to the right occurs when there is volume overload, as in aortic and mitral regurgitation. There is decreased contractility and increased compliance. The ventricle dilates to accommodate the increased volume. Hypertrophy of the cardiac muscle does not occur. Increased sympathetic activity, catecholamine administration or exercise would shift the pressure–volume curve to the left.

26

Page 27: Cardiology question

A patient who has been inadvertently given an intravenous injection of potassium chloride, develops ventricular tachycardia. His pulse is 150 beats per minute and blood pressure 60/40 mmHg.

What would be the best line of treatment in this case?Lidocaine Insulin 10 units and 50 ml of 50% glucose Amiodarone DC cardioversion 10 ml of 10% calcium gluconate

Since the patient is haemodynamically compromised, the emergency step would be to carry out DC cardioversion as utherwise there is a risk of death. Injection of 10 ml of 10% calcium gluconate would help to protect the myocardium against hyperkalaemia. Calcium ions protect the cell membranes from the effects of hyperkalaemia but do not alter the potassium concentration. Insulin drives potassium into the cell and must be accompanied by glucose to avoid hypoglycaemia. Lidocaine and amiodarone are only useful in stable cases.

A 65-year-old man with chronic renal failure has a serum potassium level of 7.1 mmol/l (normal 3.5-5.5 mmol/l).

What would be the most characteristic finding on ECG?Reduced P waves Prolonged QT intervals Prominent U waves Narrow QRS complexes T-wave inversion

Hyperkalaemia causes hyperpolarisation of cell membranes, leading to decreased cardiac excitability, hypotension, bradycardia and eventual asystole. The ECG shows characteristic tall, peaked T waves with widened QRS complexes. There is a progressive diminution in the amplitude of the P wave, which eventually disappears. Prominent U waves are seen in hypokalaemia while T-wave inversion occurs in ischaemic heart disease. Prolonged QT intervals are seen in acute myocardial infarction, hypocalcaemia, hypothermia and procainamide administration.

A patient with acute inferior wall myocardial infarction develops shock.

Which of the following complications of his MI is most likely to be the cause? Cardiac rupture Interventricular septal perforation Papillary muscle rupture Right ventricular infarction Atrial fibrillation

Right ventricular infarction occurs in one-third of cases of inferior wall myocardial infarction, which leads to pooling of blood in the right ventricle and a consequent decreased preload in the left ventricle. Hypovolaemic shock results. Papillary muscle rupture and atrial fibrillation are other complications which are also more common in inferior myocardial infarction, interventricular septal rupture is commoner in anterior myocardial infarction. Atrial fibrillation occurs as a complication in 10% of patients with myocardial infarction. It does not give rise to shock.

27

Page 28: Cardiology question

A 69-year-old man presents with a 3-hour history of chest pain. ECG shows an inferior wall infarction with ST elevation of 3 mm. There is no history of diabetes mellitus, injury or previous surgery. Blood pressure is 132/70 mmHg with a pulse of 58/min.

Which of the following treatments would be most appropriate? Tissue plasminogen activator Aspirin 2b3a inhibitor Heparin Metoprolol

Many large trials have shown that thrombolysis within 12 hours reduces the extent of ventricular damage and the mortality rate. Tissue plasminogen activator (TPA) achieves higher reperfusion rates but may be associated with a higher risk of stroke. TPA tends to be given in preference to streptokinase in patients under 50 years of age with anterior wall myocardial infarctions where the blood pressure is low (systolic < 100 mmHg), and in those patients who have previously received streptokinase. TPA also appears to be more effective than streptokinase if it is administered within 4 hours of the onset of chest pain. Intravenous heparin may be given after the initial thrombolytic therapy though its role is doubtful. Aspirin (300 mg tablet) is usually recommended. Following initiation of thrombolysis, an intravenous -blocker such as metoprolol is given, especially if the heart rate is > 100 beats per minute with persistent pain.

A 72-year-old diabetic man is admitted to A&E with a 40 minute history of central, crushing chest pain. The pain eases after an hour with bedrest, oxygen and morphine. ECG shows mild anterior ST flattening. The troponin T level is slightly raised.

What would be the optimal management of the underlying cause of his chest pain be besides usual medical measures?Discharge home with referral to the outpatients department Low molecular weight heparin Clopidogrel Thrombolysis with tissue plasminogen activator Urgent coronary angiography

This patient falls into the high-risk category of acute coronary syndrome (also called ‘unstable angina’ or ‘myocardial infarction without ST-segment elevation’). Acute coronary syndrome is a medical emergency, which, if untreated, will progress to myocardial infarction in over 10% of cases. High-risk patients may have one or more of the following features:

Prolonged, ongoing (> 20 min) rest pain Pulmonary oedema or angina with hypotension Rest angina with dynamic ST changes > 1 mm Raised troponin (I and T) Age > 70 years Diabetes mellitus

Those at high-risk should proceed promptly to angiography with a view to proceeding to revascularisation, where appropriate, during that admission.

A 67-year-old man is admitted with chronic congestive heart failure.

Based on this history, what is the most important factor to be kept in mind when prescribing drugs for this patient? Loop diuretic administration would result in a decrease in mortality Digoxin is more effective than ACE inhibitors in providing symptomatic relief Administration of a b-blocker reduces the time spent in hospital

28

Page 29: Cardiology question

Administration of spironolactone has no effect on the incidence of sudden cardiac death Angiotensin II-receptor antagonists have a better response rate than ACE inhibitors

Beta-adrenoceptor blocking agents (metoprolol, bisoprolol and carvedilol) have been found to be useful in patients with chronic stable heart failure. The studies MERIT and CIBIS 2, using the b-blockers metoprolol and bisoprolol, respectively, have shown improved symptomatic class, exercise tolerance, left ventricular function and reduced mortality in heart failure of any cause. The rapid decrease in symptoms reduces the time spent in hospital.Diuretic administration is associated with a rapid decrease in symptoms, but mortality rates are unchanged. Angiotensin-converting enzyme (ACE) inhibitors and diuretics are recommended in all patients with clinical heart failure as ACE inhibitors reduce mortality rates by 20%. Spironolactone greatly reduces the mortality and sudden cardiac death rates and should be added to the treatment. A recent trial comparing an angiotensin II-receptor antagonist (losartan) with an ACE inhibitor (enalapril) has shown no benefit of the former over the latter. Angiotensin II-receptor antagonists should be used when ACE inhibitors are contraindicated or cause side-effects (eg persistent cough).

A 3-month-old boy with a cyanotic heart lesion is found to have a patent ductus arteriosus (PDA).

What is the best treatment for maintaining patency the PDA prior to surgery?Indometacin Surgical ligation Angiographic ligation of the pulmonary artery Prostaglandin E1 administration No treatment

The ductus arteriosus in neonates and infants is highly sensitive to vasodilatation by PGE1. Patency of the ductus is necessary in patients with cyanotic heart disease until surgical correction of the heart problem is undertaken. This will ensure additional oxygenation of the blood. Administration of PGE1 has been found to be highly effective in such cases.

A 64-year-old man with Wolff–Parkinson–White syndrome presents with uneasiness and palpitations. The ECG shows fine oscillations of the baseline and no clear P waves. The QRS rhythm is rapid and irregular. The ventricular rate is 120 beats per minute. His blood pressure is 90/60 mmHg.

Which of the following interventions would be most appropriate in this case? Digoxin Verapamil DC Cardioversion Metoprolol Procainamide

This patient most probably has atrial fibrillation superimposed on WPW syndrome. The aim of treatment is to suppress the conduction ability of the abnormal pathway. This is achieved by using class-I and -III antiarrhythmic drugs but not by verapamil and digoxin, which may allow a higher rate of conduction over the abnormal pathway and precipitate ventricular fibrillation. Thus neither verapamil nor digoxin should be used to treat atrial fibrillation associated with WPW syndrome.Previous guidelines suggested that use of adenosine was an acceptable option in these patients, but now D/C cardioversion in unstable situations is seen as the intervention of choice, with procainamide an alternative.

A patient attending the cardiology clinic requires dental treatment.

Which of the following conditions merit antibiotic prophylaxis? Atrial septal defect

29

Page 30: Cardiology question

Hypertrophic cardiomyopathy Patent ductus arteriosus All of the above None of the above

Patent ductus arteriosus carries a high risk of endocarditis, but there is no robust evidence that antibiotic prophylaxis reduces the risk. The other ‘high-risk’ lesions are small ventricular septal defects and aortic regurgitation. The risk of endocarditis is highest where there are high-velocity jets of blood that damage the endothelium. Hypertrophic cardiomyopathy may be associated with high–velocity flow in the left ventricular outflow tract (LVOT) when there is marked LVOT obstruction, although, in practice, the risk of endocarditis is small. Atrial septal defects (ASDs) are large holes in a ‘low-pressure’ system and therefore carry a low risk of endocarditis and do not normally require prophylaxis. Mitral valve prolapse only carries appreciable risk where there is associated mitral regurgitation. Previous recommendations for antibiotic prophylaxis prior to dental procedures or instrumentation of the GI/GU tracts have been withdrawn as there is little evidence that these have been effective in preventing infection. Endocarditis may follow transient bacteraemia with organisms from oral or other mucosal flora, but this is much more likely to result from normal daily activity (chewing, brushing teeth) than from a visit to the dentist and antibiotic prophylaxis for such events is impractical and unnecessary.

A 19-year-old patient with Wolff–Parkinson–White syndrome presents to the emergency department with a two-hour history of rapid palpitations. The heart rate is 180 beats/min (bpm). Blood pressure is 130/80 mmHg. The electrocardiogram (ECG) shows a regular broad complex tachycardia.

What is the best treatment for the tachycardia? Direct current (DC) cardioversion Intravenous adenosine Intravenous amiodarone Intravenous flecainide Intravenous verapamil

WPW syndrome presents with atrioventricular re-entrant tachycardia caused by an aberrant re-entry circuit via the atrium, atrioventricular (AV) node and ventricle. Orthodromic tachycardia occurs most commonly and is associated with narrow complex SVT, this occurs when conduction goes via the AV node to the ventricles and then back via the accessory pathway. Antidromic tachycardia due to conduction from the atria directly to the ventricle via the accessory pathway and is associated with broad complex tachycardia. DC cardioversion is recommended for antidromic tachycardias. Treatment of orthodromic tachycardia aims to block the aV node and cut the re-entry circuit, thereby restoring sinus rhythm. The drug of choice here is adenosine. Flecainide is the drug of choice for atrial fibrillation in WPW syndrome. Cardioversion is only required when drug therapy fails or the patient's condition is compromised.

A 67-year-old lady, post-myocardial infarction, is suspected to have a left ventricular apical thrombus.

What is the most suitable imaging technique for confirming this diagnosis? Cardiac MR Left ventricular angiography Multiple uptake gated acquisition scanning Transoesophageal echocardiography Transthoracic echocardiography

Although an excellent technique for imaging the posterior cardiac structures (atria, left atrial appendage, valves and pulmonary veins), transoesophageal echo is less useful for imaging the structure and function of the ventricles, especially the left ventricular apex, which is better imaged by conventional two-

30

Page 31: Cardiology question

dimensional (2D) echo. Atypical thrombus may be apparent on contrast left ventriculography, but this technique carries a risk of dislodging and embolising interventricular thrombus.

A 30-year-old postman with hypertension but normally in good health presents to the emergency department with sudden severe breathlessness and sweating. Chest examination reveals bilateral basal crackles. He improves with diamorphine and frusemide (furosemide). Electrocardiograms (ECGs) and cardiac enzymes are normal. He develops two further episodes of pulmonary oedema which respond well to diuretics.

The most likely cause of pulmonary oedema is? Dilated cardiomyopathy Myocarditis Ischaemic heart disease Phaeochromocytoma Renal artery stenosis

The differential diagnosis of acute pulmonary oedema includes severe left ventricular (LV) dysfunction, paroxysmal arrhythmias, three-vessel or left main stem coronary disease and, in the context of hypertension, renal artery stenosis and phaeochromocytoma. Renal artery stenosis is more common. This man is a postman and therfore would be expected to have reasonable exercise tolerance, given his age this makes cardiomyopathy, myocarditis and ischaemic heart disease extremely unlikely. Renal artery stenosis in a 30-year-old is much more common than phaeochromocytoma.

A 46-year-old Asian man with a past history of coronary artery bypass grafting presents with breathlessness. The jugular venous pressure (JVP) shows prominent x and y descents.

The most likely cause is? Constrictive pericarditis Dilated cardiomyopathy Pericardial effusion Restrictive cardiomyopathy Severe mitral regurgitation

A prominent x descent in the jugular venous pressure (JVP) may occur in constrictive pericarditis or pericardial effusion. The y descent is lost in tamponade but prominent in constrictive pericarditis. Constrictive pericarditis was classically caused by tuberculosis, but today is more commonly associated with cardiac surgery, renal failure or following infective pericarditis. Restrictive cardiomyopathy may produce clinical features similar to constriction, but is less common.

A neonate is noted to be cyanosed within the 24 h following delivery.

Which cardiac abnormality would be the most likely cause? Ebstein’s anomaly Eisenmenger ventricular septal defect Hypoplastic left heart Tetralogy of Fallot Transposition of the great vessels

Although tetralogy of Fallot is much more common than transposition, the right ventricular (RV) outflow tract gradient, which is the major determinant of cyanosis in Fallot’s, does not become maximal until 6–9 months after birth. Many babies with Fallot’s are, therefore, pink at birth but gradually become cyanosed over the first few months of life. Ebstein’s anomaly is a congenital abnormality of the tricuspid valve associated with in utero exposure to lithium, which does not normally cause cyanosis.

31

Page 32: Cardiology question

A 49-year-old man is noted to have shortening of the QT interval on the ECG.

Which drug is most likely to be responsible?Amiodarone Atenolol Digoxin Flecainide Sotalol

The cardiac glycosides (digoxin and ouabain) shorten the QT interval. Class Ia (eg disopyramide), class Ic (eg flecainide) and class III drugs (eg amiodarone and sotalol) all prolong the QT interval. -blockers have a neutral effect on the QT interval but are effective at stabilising the QT in long QT syndromes.

A 60-year-old woman is found to have a systolic murmur at a routine medical. When you see her she is asymptomatic. Electrocardiography (ECG) shows marked left ventricular hypertrophy with strain. Echocardiography shows a peak aortic valve gradient of 90 mmHg, and decreased LV systolic function.

What is the correct management?Aortic valvuloplasty Anticoagulation Regular out-patient review Routine aortic valve replacement Urgent aortic valve replacement

Surgery for aortic valve replacement is indicated in symptomatic patients (angina, exertional breathlessness, syncope) as the risk of sudden death increases dramatically with the onset of symptoms, or, in those with severe asymptomatic disease (peak outflow gradient greater than around 50 mmHg). Patients with a gradient of less than 25 mmHg have a 20% chance of needing surgical intervention within 15 years. Valvuloplasty is used only in patients with critical aortic stenosis who are unfit for surgery as the benefits are usually short-lived.

A 67-year-old man with chronic heart failure is reviewed in terms of his drug therapy.

Which of the following treatments has no proven mortality benefit? Bisoprolol Digoxin Enalapril Nitrates and hydralazine Spironolactone

Bisoprolol (CIBIS II), spironolactone (RALES), enalapril (CONSENSUS) and nitrates and hydralazine (V-HEFT) have all been shown to improve mortality in chronic heart failure. Digoxin reduces the risk of death due to heart failure but overall cardiovascular mortality is similar to that on placebo, probably reflecting a small increase in the risk of arrhythmic death with digoxin therapy.

What is the most likely lipid abnormality in a 48-year-old Asian man with good glycaemic control?

Elevated high-density lipoprotein (HDL) Elevated low-density lipoprotein (LDL) Elevated LDL/elevated triglycerides Low HDL/elevated LDL Low HDL/elevated triglycerides

32

Page 33: Cardiology question

Asians do not have classical LDL–related risk for ischaemic heart disease. Their profile includes low HDL and elevated triglycerides, meaning that measurement of LDL alone may underestimate their risk.

56-year-old man has known tricuspid regurgitation.

Which part of the jugular venous waveform is likely to be most prominent? a wave c wave v wave x descent y descent

Tricuspid regurgitation characteristically causes loss of the x descent in the JVP causing fusion of the c and v waves to produce ‘giant’ v waves, also known as ‘cv’ waves or ‘s’ waves.

A 56-year-old lady has a known ventricular septal defect.

Which of the following clinical signs would most indicate the presence of established pulmonary hypertension? Loud systolic murmur Raised jugular venous pressure (JVP) Single loud second heart sound Systolic thrill Displaced apex beat

The systolic murmur and thrill of the ventricular septal defect (VSDs) are absent once Eisenmenger's complex has developed. Under these circumstances there are just signs of pulmonary hypertension and cyanosis. Cardiomegaly may occur with Eisenmenger's complex because of right venticular (RV) enlargement but the left ventricle (LV) is not usually significantly enlarged and the apex beat is not typically displaced. A single second heart sound is characteristically associated with Fallot’s tetralogy where pulmonary stenosis protects from developing pulmonary hypertension.

A 45-year-old man with a strong family history of ischaemic heart disease presents with atypical chest pains. Electrocardiographic (ECG) exercise testing shows J point depression of 1 mm with a heart rate of 120 beats/min (bpm).

What is the most appropriate next step? Coronary angiography Dobutamine stress echocardiography Radionuclide myocardial perfusion scanning Reassure and discharge Repeat ECG exercise testing on anti-anginal medication

J point depression is a physiological response to an increase in heart rate and ST segments should, therefore, be measured 80 ms post-J. J point depression produces upward sloping ST depression which has little predictive value for coronary artery disease. Horizontal and downward sloping segments are more predictive than upward sloping ones. In a young man with atypical chest pain, this exercise test would be reassuring and there would be no indication for further investigation or treatment.

A 56-year-old man presents with a cardiac rhythm disorder.

33

Page 34: Cardiology question

Which one of the following scenarios would be an indication for temporary transvenous cardiac pacemaker insertion? Asymptomatic 2.8 s sinus pauses A short period of complete heart block complicating inferior myocardial infarction, (pre-thrombolysis) with blood pressure 110/70 mmHg Asymptomatic complete heart block with broad complex ventricular complexes at 35 bpm Mobitz II AV block complicating anterior myocardial infarction with blood pressure 110/70 mmHg Bifascicular block prior to aortic aneurysm repair

Asymptomatic sinus node disease, even with prolonged pauses, carries a low risk of sudden death and can usually be managed without a temporary wire. Chronic complete heart block has a higher risk but if asymptomatic and with a stable escape rhythm can also be managed in most cases without a temporary wire. Bifascicular block carries a slightly higher risk of high–grade AV block but, in asysmptomatic patients, this risk is sufficiently low to obviate the need for perioperative pacing. In myocardial infarction, AV block and even complicating MI should be managed conservatively if asymptomatic and causing no haemodynamic compromise. However, second- or third-degree heart block complicating anterior MI requires pacing.

A 15-year-old patient undergoes echocardiography, which shows a right-sided aortic arch.

Which cardiac condition is most likely? Coarctation of the aorta Ebstein’s anomaly Tetralogy of Fallot Hypoplastic left ventricle Noonan’s syndrome

Approximately 25% of patients with Fallot’s tetralogy have a right–sided aortic arch. Coarctation is associated with bicuspid aortic valve and Noonan’s syndrome with pulmonary stenosis. Ebstein’s anomaly is a congenital abnormality of the tricuspid valve which has an association with right–sided accessory pathways.

A 70-year-old man undergoes successful DC cardioversion for atrial fibrillation (AF).

Which one of the following factors best predicts long-term maintenance of sinus rhythm following this procedure? Age under 75 years Normal left ventricular function Warfarin therapy No alcohol intake AF duration less than 6 months prior to cardioversion

Cardioversion has a much higher success rate in patients with structurally normal hearts but the left atrial size is a better predictor than left ventricular function. AF is likely to be persistent where the left atrial dimension is >5 cm. Age is much less important than the duration of AF. The success of cardioversion drops off significantly after 6 months of persistent AF and long-term sinus rhythm is unlikely to be restored if AF has been persistent for more than 12 months. Alcohol is an important aetiological factor but less significant than AF duration. Warfarin is important to reduce stroke risk but does not help to restore or maintain sinus rhythm.

A 38-year-old man presents for review. His only previous history of note has been recurrent shoulder subluxation. His main complaints are tiredness and increasing dyspnoea on exertion. The nursing clerking on admission notes that he seems very tall and thin, his height is described as 1.93 m (6ft 4 inches). On

34

Page 35: Cardiology question

examination his blood pressure is 165/70 mmHg, he has left ventricular hypertrophy, a low-pitched apical diastolic murmur and an early systolic apical ejection murmur.

What diagnosis fits best with this clinical picture? Mitral stenosis Aortic regurgitation Mitral valve prolapse Aortic stenosis Infective endocarditis

This man has a marfanoid habitus and is at risk of suffering aortic regurgitation. Aetiological factors involved in aortic regurgitation include infective endocarditis, rheumatic heart disease, trauma with valvular rupture, congenital bicuspid aortic valve, myxomatous degeneration, syphilitic aortitis, systemic lupus erythematosus (SLE), aortic dissection and the use of amphetamine slimming products. Symptoms of aortic regurgitation include dyspnoea on exertion, syncope, chest pain and congestive heart failure.Cardiac auscultation characteristically reveals displacement of the cardiac impulse downwards and to the left, prominent S3 heard over the apex, a low-pitched apical diastolic rumble (Austin–Flint murmur) and an early systolic apical ejection murmur. Chest X-ray may reveal left ventricular hypertrophy and aortic dilatation. Echocardiography reveals the coarse diastolic fluttering of the anterior mitral valve leaflet.Surgical valve replacement is indicated in symptomatic patients with chronic aortic regurgitation who have symptoms despite optimal medical management, and in acute aortic regurgitation where there is evidence of left ventricular failure. Ideally, surgery should be considered before the ejection fraction falls to below 55%.

An 82-year-old man was admitted to the Emergency department from a local church service. He fainted and another parishioner, who is a trained first-aider, reported that he was pulseless for a few seconds after the attack. On examination his blood pressure was 165/95 mmHg (past history of hypertension), he had no murmurs on auscultation of the chest and carotid auscultation was also normal. Outpatient 7-day ambulatory cardiac rhythm monitoring was arranged, which is now reported as normal.

What diagnosis fits best with this presentation? Transient ischaemic attack Transient bradycardia Paroxysmal atrial fibrillation Simple syncope Carotid sinus syndrome

The incidence of carotid sinus syndrome is said to be around 10% in the adult population. This incidence increases with age, and men are affected twice as often as women. Presentation is rare below the age of 50 years. The incidence also increases in patients with hypertension, but often a definitive cause may not be identified. Predisposing factors may include head and neck tumours, neck surgery, significant lymphadenopathy or carotid body tumours. Physical examination in this case suggests that there is no significant cardiac pathology, and the 7-day Holter monitor result makes arrhythmia less likely as a cause.Supine carotid sinus massage with blood pressure and ECG monitoring is the diagnostic procedure. It should not be performed in patients with a history of cerebrovascular disease or carotid bruits, and should only be applied to one artery at a time. The response may be cardioinhibitory with asystole for at least 3 s, or vasopressor with a drop of more than 30 mmHg (in the presence of symptoms) or more than 50 mmHg without symptoms. A mixed picture of a cardioinhibitory and vasopressor response may occur. Pacemaker insertion is recommended for patients with cardioinhibitory carotid sinus syndrome.

A 22-year-old student is admitted by ambulance from a local night club. He has no previous medical history of note and is adopted so is unaware of his family history. Bystanders who have accompanied him say that he suffered sudden collapse while dancing. Bouncers at the club claim that they couldn’t feel a strong pulse during

35

Page 36: Cardiology question

his period of unconsciousness. On admission his blood pressure is 120/60 mmHg, and pulse is 80 bpm and regular. ECG looks normal, corrected QT interval is 0.6 s.

What diagnosis fits best with his clinical picture? Simple syncope Long QT syndrome – mutation uncharacterised Ecstasy overdose Carotid sinus syndrome Jervell–Lange-Nielsen (JLN) syndrome

This man’s QT interval is prolonged. JLN syndrome is also associated with long QT, but patients have deafness in addition to the cardiac rhythm abnormality. Episodes of severe QT prolongation and torsades de pointes ventricular tachycardia in congenital long QT syndrome may be precipitated by increased adrenergic drive (such as that from dancing in a night club). This patient is adopted, so that it may be possible that there is an unknown family history of sudden death. The molecular biology of long QT syndromes is heterogeneous, and a number of different mutations coding for potassium or sodium channels may be responsible.Where specific mutations are identified, antiarrhythmic therapy may be specifically targeted to provide optimum therapy. In patients who respond poorly to medical treatment, implantable defibrillator may be considered.

You review a 61-year-old man with paroxysmal atrial fibrillation. You consider a class Ic antiarrhythmic agent as the most appropriate choice to maintain him in sinus rhythm.

What statement best describes the effect that class Ic agents have on electrical activity in the heart? Lengthen the cardiac action potential Shorten the cardiac action potential Widen the duration of the action potential Have minimal effect on the duration of the action potential Predominantly affect the AV node

Class Ic antiarrhythmic agents such as flecainide or propafenone have no significant effect on the cardiac action potential. They are commonly used for the treatment and prophylaxis of atrial arrhythmias such as paroxysmal atrial fibrillation and atrial tachycardia.Class Ia agents such as quinidine lengthen the action potential, Ib agents such as lidocaine shorten the action potential, and class III agents widen the duration of the action potential. Beta-blocking agents predominantly affect the sinus node, whereas calcium-channel blocking agents predominantly affect the atrioventricular (AV) node.It is important to note that class I agents should be avoided in patients with significant coronary artery disease; flecainide, in particular, was shown to be associated with increased mortality in a postmyocardial infarction study.

A 26-year-old woman attends her GP for an insurance medical. Her previous medical history is unremarkable. On examination her BMI is 21, blood pressure is 105/62 mmHg, and auscultation of the heart reveals a late systolic click, and a late diastolic murmur (these findings being accentuated in the standing position).

What diagnosis fits best with this clinical picture? Atrial septal defect Mitral regurgitation Mitral stenosis Mitral valve prolapse Constrictive pericarditis

Mitral valve prolapse is the posterior bulging of leaflets of the mitral valve in systole. It is thought to be present in around 4% of the population, with a higher incidence in females. Increased incidence is

36

Page 37: Cardiology question

associated with autoimmune thyroid disease, Ehlers–Danlos syndrome, Marfan’s syndrome, pseudoxanthoma elasticum and pectus excavatum.Clinically, patients are often young females with a narrow anteroposterior (AP) chest diameter, low body weight and low/normal blood pressure. Cardiac auscultation reveals a mid to late systolic click, best heard at the apex, and a mid to late diastolic murmur with findings accentuated in the standing position. Associated embolic phenomena (stroke or transient ischaemic attack (TIA)) are rare. Echocardiography reveals bulging of the anterior and posterior mitral valve leaflets in systole.The incidence of complications of mitral valve prolapse is thought to be less than 1% per year, and treatment is often not required. It is, however, key to consider prophylaxis against bacterial endocarditis, as the risk is put at between three and eight times that of the general population.

A 72-year-old man presented with an episode of collapse. He had experienced two similar episodes recently, each lasting about one minute. Four years previously he suffered an anterior myocardial infarction. On examination he was orientated and symptom-free with a regular pulse rate of 80 beats per minute (bpm), blood pressure 140/80 mmHg, and the apex beat was displaced to the left. There was an apical systolic murmur. There were no signs of trauma. The electrocardiogram (ECG) showed sinus rhythm, Q waves, and ST segment elevation anteriorly without reciprocal depression.

What is the diagnosis? Acute anterior myocardial infarction Cerebrovascular accident Epileptic seizure Pulmonary embolism Ventricular tachycardia

It is likely that this man has suffered a transient episode of ventricular tachycardia, which results in a period of circulatory compromise that leads to the collapse. The persistent ST segment elevation in this case would not indicate acute myocardial infarction, but it is likely to represent left ventricular aneurysm, a recognised complication of acute anterior myocardial infarction. Holter monitoring of his electrocardiogram (ECG) would be the investigation of choice –seven-day cardiac monitors are now available and would indicate the best chance of capturing a period of ventricular tachycardia. Anti-arrythmic of choice would be amiodarone, although this patient should probably be considered for an implantable defibrillator once the diagnosis is confirmed.

A 58-year-old male patient has suffered from a recent acute myocardial infarction 3 days ago. He becomes acutely unwell with a hypotensive episode. There is a pansystolic murmur which is accentuated by inspiration, along the lower left sternal border. A Swan-Ganz catheter was inserted and the following was noted: right atrial pressure was 12 (very high); calculated left atrial pressure was 2 (low normal).

What is the likely cause? Right heart failure Left heart failure Mitral regurgitation Tricuspid regurgitation Aortic regurgitation

Tricuspid regurgitation may occur in post-myocardial infarction, in association with cor pulmonale, rheumatic heart disease, infective endocarditis, carcinoid syndrome, Ebstein’s anomaly, and other congenital abnormalities of the atrioventricular valves. Regurgitation gives rise to high right atrial pressures (as seen here). Physical signs include a large jugular venous cardiovascular wave and a pulsatile liver that pulsates in systole. A right ventricular impulse may be felt at the left sternal edge and there is a blowing pansystolic murmur. Severe tricuspid regurgitation may require valve repair, or rarely replacement. Another consideration with this type of presentation post-myocardial infarction is pulmonary embolus, a high proportion of those patients who die post-myocardial infarction, do so because of thrombo-embolic disease.

37

Page 38: Cardiology question

A patient presents with shortness of breath and ankle swelling. An echocardiogram has been ordered to determine the left ventricular ejection fraction.

Which echocardiography mode is the most appropriate? M-mode A-mode Modern transthoracic Continuous wave Power wave

Modern transthoracic echocardiography combines real-time two-dimensional imaging of the myocardium and valves with information about velocity and direction of blood flow obtained by Doppler and colour flow mapping. It is non-invasive and a complete examination can be performed in most patients in less than 30 min. M-mode echocardiography has preceded modern two-dimensional imaging. Unlike two-dimensional imaging, which uses a series of sweeps across the heart, M-mode uses a single static beam of very frequent ultrasound pulses. The narrow beam is analogous to a vertical mineshaft passing through various layers of rock. Displayed in real time, this results in reflections from cardiac structures being displayed as horizontal lines with superficial structures at the top of the screen and the deeper structures at the bottom. This data are interpretable when one knows which structure each line represents, and the technique has excellent spatial resolution.

A patient with left ventricular failure undergoes echocardiography.

Which is the correct formula for calculating the ejection fraction (EF)? EF = [end diastolic volume (EDV) – end-systolic volume (ESV)]/EDV EF = [end diastolic volume (EDV) – end-systolic volume (ESV)]/heart rate (HR) EF = [heart rate (HR) × end diastolic volume (EDV)]/end-systolic volume (ESV) EF = [heart rate (HR) × end-systolic volume (ESV)]/end diastolic volume (EDV) EF = [end-systolic volume (ESV) – end diastolic volume (EDV)]/EDV

Ejection fraction (EF) is calculated using the following equation:EF = [end diastolic volume (EDV) - end-systolic volume (ESV)] / EDV

A 69-year-old man has been admitted to the emergency department with syncope. He felt hot, complained of nausea and then fainted. His electrocardiogram (ECG) was normal. His brother suffers from adult onset epilepsy.

What is the most appropriate investigation? Electroencephalogram (EEG) 24-h ECG Computed tomography (CT) of the brain Echocardiography Tilt test

The development of tilt testing has allowed the study of the pathophysiology of neurocardiogenic syncope. The patient is strapped to a tilt-table and is tilted, head upright, usually at 70 degrees for up to 45 min. Protocols that use additional provocation with isoprenaline or nitrates are also commonly used. Blood pressure and cardiac rhythm are monitored throughout the tilt test. In neurocardiogenic syncope, the patient classically maintains normal blood pressure initially, until the sudden onset of syncope is associated with severe hypotension and bradycardia, often preceded by tachycardia. These features resolve with return to the supine posture. Some patients have a mainly vasodepressor response, with

38

Page 39: Cardiology question

hypotension and little change in heart rate, while others have a marked cardioinhibitory response, with severe bradycardia or asystole of several seconds’ duration. However, most patients exhibit a mixed response, and those patients with marked cardioinhibition also have a preceding vasodepressor response. This is an important observation when treatment is considered, since permanent pacing to maintain cardiac rhythm may not cure all symptoms, because falls in blood pressure may still occur even when bradycardia is prevented. In this case the factor of the family history of adult-onset epilepsy is irrelevant given the clinical picture of syncope.

Which of the following statements are most indicative of myocardial ischaemia?

Associated shortness of breath Claudication Dizziness Radiation to jaw Relief by glyceryl trinitrate

The pain of angina usually radiates out from the chest, the commonest sites of radiation include the following.

The neck and throat, causing a feeling of choking, strangulation or suffocation. The jaw, and maybe interpreted as toothache or problems with dentures. Down one or both arms: this is usually felt down the inside, under the axilla to the inner two

fingers. By contrast muscular pain usually runs over the shoulder and down the outside of the arm.

Other sites include the abdomen, the back, and areas of previous injury. Angina is often misinterpreted as indigestion.

Which is the best clinical marker of the severity of aortic stenosis?

Character of apex beat Character of carotid pulse Character of S2 Intensity of murmur Pulse rate

Physical findings of aortic stenosis may include a narrow pulse pressure, especially when stroke volume decreases, and a slow-rising, small-volume carotid pulse. However, the poorly compliant arterial wall may mask these abnormalities, so that the carotid pulse appears relatively normal. The cardiac apex impulse is forceful and sustained, but this finding may be masked by kyphosis (in which the anteroposterior diameter of the chest is increased). The first heart sound is soft. The aortic component of the second heart sound is also soft; it may be inaudible when stenosis is severe and the valve is heavily calcified. Reverse splitting of the second heart sound may occur in patients with left ventricular failure. A fourth heart sound is common but disappears in one-quarter of elderly patients who develop atrial fibrillation. Ejection sounds are rare because the valve cusps are immobile.

A 65-year-old male patient with stable angina complains of shortness of breath after walking two flights of stairs. He has normal left ventricular function on the echocardiogram and a positive exercise tolerance test (3 mm ST depression at stage III).

What is the most appropriate therapy? Atenolol Simvastatin Isosorbide mononitrate Angiotensin-converting enzyme (ACE) inhibitor

39

Page 40: Cardiology question

Nicardipine

Beta-blocking agents are the cornerstone of the pharmacological management of chronic angina pectoris. They are well tolerated and reduce the frequency and duration of anginal episodes and improve exercise tolerance. They are also effective antihypertensive agents and prevent some arrhythmias. They act by competitively inhibiting catecholamine effects on the b-adrenergic receptor. This reduces heart rate and improves coronary perfusion (by prolonging diastole), thereby reducing an exercise-induced rise in blood pressure and contractility.

A 65-year-old female patient with severe heart failure presents with increasing shortness of breath. Her current pharmacological treatment consists of an angiotensin-converting enzyme (ACE) inhibitor, loop diuretic and b-blocker.

What is the most appropriate management? Add digoxin Add spironolactone Stop b-blocker Stop ACE inhibitor Add simvastatin

The positive inotropic effects of cardiac glycosides can be useful in reducing symptoms (mainly breathlessness) in patients already taking diuretics and angiotensin-converting enzyme (ACE) inhibitors, although they have no effect on mortality and must be used with caution in patients such as this who are prone to disturbances of potassium. Low dose spironolactone has however been shown to have positive effects on cardiovascular mortality in patients with end stage heart failure. Large outcome studies with carvedilol and bisprolol have also shown a mortality benefit in heart failure.

A 22-year-old cocaine addict presents with an acute myocardial infarction. His blood pressure is 180/110 mmHg.

Which is the most appropriate treatment? Thrombolysis HeparinPercutaneous coronary interventionNaloxoneGlycoprotein 2b/3a inhibitors

Cocaine use has recently been implicated as a cause of unstable angina. Three possible mechanisms by which cocaine induces myocardial ischemia are: (1) increased myocardial oxygen demand, (2) decreased myocardial oxygen supply secondary to vasospasm or coronary thrombosis, and (3) direct myocardial toxicity. Documented cocaine use should not be considered to rule out underlying significant coronary artery disease (CAD), since the drug may precipitate coronary vasospasm or acute myocardial infarction in the patient with atherosclerotic CAD.Where elective angioplasty is available, this is preferable to thrombolysis as outcome studies show it to be superior.

A 32-year-old woman who is known to be 17 weeks’ pregnant presents for review. She has periods of paroxysmal supraventricular tachycardia (SVT) and on this occasion has a ventricular rate of 165/min and a blood pressure of 90/50 mmHg, feeling faint and unwell.

Which of the following anti-arrhythmics would be the most appropriate prophylaxis for her?FlecainideAmiodaroneDigoxin

40

Page 41: Cardiology question

PhenytoinPropafenone

This patient has paroxysmal supraventricular tachycardia (SVT). While digoxin slows the ventricular rate in patients with chronic atrial fibrillation, it does not maintain sinus rhythm in patients with paroxysmal tachycardia. Amiodarone is known to be teratogenic and is contraindicated in pregnancy. While there is no evidence that flecainide is teratogenic, no randomised controlled trials have included pregnant women, for obvious reasons. Several case series describe use of flecainide in pregnant women and it appears to have a relatively good safety profile compared with other anti-arrhythmics.

You are asked to review a 19-year-old woman who presents with increasing shortness of breath on exercise. She is from a travelling family and has rarely encountered medical care. On examination she appears of short stature with extra skin folds around her neck, and appears to have failure of secondary sexual development. Her blood pressure is raised at 165/100 mmHg. She reports that her legs feel tired all the time and she has occasional chest pain on exercising.  Which of the following cardiac diagnoses fits best with her clinical condition? Pulmonary stenosisMitral regurgitationCoarctation of the aorta Aortic regurgitationHypertrophic obstructive cardiomyopathy

Given this patient’s clinical picture it appears that she has Turner’s syndrome. It is likely that, because she is from a travelling family, she has only had irregular access to medical services. Turner’s syndrome is associated with coarctation of the aorta, and bicuspid aortic stenosis may also co-exist. Symptoms include vigorous pulsation in the neck or throat, hypertension, tired legs or intermittent claudication on running, left ventricular failure, angina pectoris, and aortic rupture. Physical signs to note include radiofemoral pulse delay, development of collateral vessels, left ventricular failure, and ejection systolic murmur. Electrocardiogram (ECG) shows left ventricular hypertrophy and right bundle branch block is common.

A 41-year-old man with a family history of sudden death presents to casualty with a second episode of collapse. On this occasion he is referred to the Cardiology Department for review. Echocardiography reveals asymmetrical septal hypertrophy, abnormal systolic motion of the anterior mitral valve leaflet and narrowing of the left ventricular outflow tract. The 24-h electrocardiogram (ECG) monitoring as an outpatient reveals several periods of non-sustained ventricular tachycardia. Which of the following would be most appropriate for the management of his arrhythmia? Oral flecainide 100 mg dailyOral amiodarone 200 mg tdsOral amiodarone 200 mg dailyImplantable cardioverter defibrillatorPhenytoin 100 mg po daily

Given this clinical history, this patient has a diagnosis of hypertrophic obstructive cardiomyopathy. He has non-sustained ventricular tachycardias and is at significant risk of sudden death. While myomectomy will impact on symptoms, it won’t affect the risk of arrhythmia. Management in the past would have been medical therapy with amiodarone, but recent studies have demonstrated superior efficacy for implantable cardioverter defibrillators (ICDs). Given that long-term use of amiodarone is associated with significant morbidity, ICDs are taking over as management of choice.

41

Page 42: Cardiology question

A 72-year-old man presents for an extraction of three teeth under local anaesthesia. He has a past history of rheumatic heart disease. Mitral stenosis has been identified but the rotten teeth are being removed before valve replacement. He is allergic to penicillin.

Which of the following would be the most appropriate antibiotic regime for him?Amoxicillin 3 g po 1 hour before procedureNo prophylaxis necessaryVancomycin 1 g po 1 hour before procedureCiprofloxacin 1 g po 1 hour before procedure Augmentin 1 g po before procedure

After an extensive evidence review, the most recent NICE guidelines do not recommend antibiotic prophylaxis for a number of routine procedures including dental extraction. Their conclusion is based on the fact that they were unable to determine any evidence which suggested that antibiotic prophylaxis prevented the development of endocarditis.

A 45-year-old man attends for review. He has been suffering increasing shortness of breath over the past few years. He is a non-smoker who drinks 20 units per week of alcohol and has no significant past cardiovascular history. Now he presents with what seems to have been a transient ischaemic attack (TIA), with weakness and co-ordination problems affecting his left side, which have resolved over the past 24 hours. On examination blood pressure is 142/95 mmHg and he is in sinus rhythm. There is no opening snap, but a diastolic murmur is heard which changes in character according to posture. Bloods are unremarkable, including C-reactive protein (CRP), which is in the normal range. Which of the following diagnoses fit best with this clinical picture?Right atrial myxomaLeft atrial myxomaAortic stenosisMitral stenosisMitral regurgitation

This patient has suffered a TIA, most likely due to embolus from an intra-cardiac cause. One possible clinical explanation could be mitral stenosis, left atrial enlargement and atrial fibrillation, leading to clot formation within the atrium, but this patient is in sinus rhythm, there is no opening snap on auscultation, and the murmur changes in character with posture. This suggests the possibility of another cause, and left atrial myxoma would fit the bill. Myxoma can occur in any cardiac chamber, but occurs most commonly in the left atrium. It is a gelatinous, friable tumour, which leads to transient signs of mitral stenosis that only occur if the tumour approaches the mitral valve orifice. There is no opening snap. There may be an early diastolic plop as the tumour prolapses through the mitral valve. X-ray may show calcification within the tumour if it is long standing. Definitive treatment involves surgical excision; recurrence rate is extremely low, but follow up is recommended for a period of 5 years. Right atrial myxomas are more rare and difficult to identify clinically; there may be evidence of multiple pulmonary infarcts due to formation of emboli.

A 75-year-old man is referred for total hip replacement. He has a history of hypertension and angina and has suffered a myocardial infarction some 8 years earlier. Current medication includes atenolol 50 mg daily, ramipril 10 mg daily, aspirin 75 mg daily and isosorbide dinitrate 60 mg. Blood pressure at the preoperative assessment was 160/80  mmHg but he maintains that his readings with the general practitioner have been normal. He last had an exercise test some 3 years earlier and managed 8 min with no significant electrocardiogram (ECG) changes. Which one of the following investigations in addition to standard assessment would be most appropriate for the preoperative assessment of this patient? Repeat exercise ECG testRoutine echocardiogram

42

Page 43: Cardiology question

99Tcm MIBI SPECT scanStress ECGMagnetic resonance angiography

From the history given it appears that this man has relatively stable angina and there seems little value to be gained from further imaging of his coronary arteries. Routine echocardiogram would add information about left ventricular function and point out any valvular disease. Given possible haemodynamic changes during total hip replacement, significant left venticule (LV) impairment or valvular disease may impact significantly on operative prognosis. His blood pressure does appear labile and it would be worthwhile to document that his normal blood pressure is within acceptable limits. If he has significant white coat hypertension, when he attends for surgery he may well be cancelled if his blood pressure is too high. For this reason, 24-h outpatient blood pressure recording would seem sensible.

A 31-year-old man presents with transient loss of consciousness and palpitations. ECG shows ventricular tachycardia (not SVT).

Which of the following cannot be used safely?Adenosine Amiodarone DC cardioversion Flecainide Verapamil

Adenosine can be used in paroxysmal supraventricular tachycardias to return the heart to sinus rhythm. It is also of use in helping to differentiate between broad complex tachycardias of supraventricular and ventricular origin. Amiodarone is a type III antiarrhythmic. It is used in the treatment of paroxysmal SVT, VT, atrial fibrillation and atrial flutter and in VF. Synchronised DC cardioversion can be used in VT after anaesthetising the patient. In pulseless VT, DC cardioversion is used as an emergency procedure. Flecainide, a type Ic antiarrhythmic can be used safely in symptomatic ventricular arrhythmias. This agent should be avoided in patients with known coronary artery disease and/or impaired left ventricular function. However Verapamil, a type IV antiarrhythmic is not for use in VT, and should be avoided in tachycardias unless a supraventricular origin has been established.

A 64-year-old woman suffers from frequent and painful urinary tract infections. After her third course of antibiotics in the past 6 months she is advised by the GP to take cranberry juice supplements. Significant past medical history of note includes hypertension for which she takes ramipril and bendroflumethiazide and hypercholesterolaemia for which she takes simvastatin. There is also a history of paroxysmal atrial fibrillation for which she takes warfarin and amiodarone. Which of her medications is most likely to interact with the cranberry juice?SimvastatinAmiodaroneBendroflumethiazideWarfarinRamipril

Cranberry juice contains a number of bioflavinoids, some of which are thought to cause inhibition of the cytochrome p450 2C9 isoenzyme which is responsible for warfarin metabolism. Metabolism of simvastatin is inhibited by grapefruit juice. The warfarin interaction was given as a Committee for Safety of Medicines (CSM) warning in 2003 and features prominently in the MRCP examination.

A 25-year-old man was found by his family at home having suffered a cardiac arrest. He was previously well, apart from well controlled Type 1 diabetes controlled with a basal bolus insulin regime. His family followed the ambulance and ask if they can be in the resuscitation room. After 20 mins of repeated resuscitation cycles he has

43

Page 44: Cardiology question

remained in asystole.

Blood gases; pH 7.01

PO2 8.4 kPa

PCO2 3.9 kPa

Bicarb 10 mmol/l

Which person is the most appropriate person to make the decision to discontinue resuscitation?A&E consultant On call medical consultant Parents of the patient Patient’s fianceé Resuscitation team leader

Clear, appropriate communication is a key component of resuscitation. Whilst it may of course be appropriate to allow the presence of relatives within the resuscitation room, they do not have the authority to continue or discontinue resuscitation; of course relatives do however need to be informed of progress. Whilst the experience of the A&E or on-call medical consultants may be useful in gaining advice, the resuscitation team leader is usually a senior anaesthetist or physician in their own right, and qualified to decide on discontinuing resuscitation.

A 70-year-old lady with a history of asthma presents with shortness of breath for some days. She is also treated with ramipril 10mg daily. On examination her blood pressure is 135/85 mmHg, pulse is 100/min (atrial fibrillation). She is not in cardiac failure. Examination of the respiratory system reveals wheeze consistent with asthma. Results; Hb 13.2 g/dl

WCC 6.1 x109/l

PLT 240 x109/l

Na+ 138 mmol/l

K+ 4.7 mmol/l

Creatinine 125 μmol/l

CXR Cardiomegaly consistent with longstanding hypertensive heart disease

Which of the following is the most appropriate treatment for her atrial fibrillation?Diltiazem Digoxin Amiodarone Atenolol Dysopyramide

This lady has been in atrial fibrillation for an unknown period of time and her pressing need at this point is rate control. Options for rate control include diltiazem, atenolol and digoxin. Atenolol may lead to worsening of her asthma and should not be used. Diltiazem is negatively inotropic and may precipitate cardiac failure; as such digoxin is an effective option for rate control here.

A 71-year-old lady with a history of one previous myocardial infarction presents to the Emergency department. She has sudden onset shortness of breath and palpitations which happened after her dinner a couple of hours earlier. A previous ECG from clinic a month earlier shows sinus rhythm. Medication includes ramipril 10mg daily, amlodipine 10mg daily and aspirin 75mg. On examination her blood pressure is 100/60 mmHg, pulse is 140/min irregular and she has evidence of LVF.

44

Page 45: Cardiology question

Bloods Hb 14.0 g/dl

WCC 6.7 x109/l

PLT 190 x109/l

Na+ 140 mmol/l

K+ 5.0 mmol/l

Creatinine 130 µmol/l

ECG Fast atrial fibrillation, lateral ST depression

Which of the following is the most appropriate medication to control her AF?Digoxin Amiodarone Flecainide Sotalol Verapamil

Flecainide, whilst effective at cardioverting atrial fibrillation is contra-indicated in patients with a history of ischaemic heart disease since the CAST post infarct trial demonstrated increased mortality in patients treated with flecainide. Sotolol and verapamil are negatively inotropic and likely to worsen LVF. Digoxin would be effective at slowing the ventricular rate, but given that she was in sinus rhythm 1 month earlier, it would be a reasonable objective to attain sinus rhythm again. As such IV loading with amiodarone would appear to be the most appropriate option in this patient.

A 78-year-old lady is admitted from home by ambulance. She was found lying on the floor by her home help after suffering a fall. She has a history of hypertension managed with ramipril 10mg PO daily. On examination her temperature is 30.0oC, her BP is 100/50 mmHg, with a pulse of 52/min. She has a fractured left neck of femur.

Bloods; Hb 14.5 g/dl

WCC 4.5 x109/l

PLT 192 x109/l

Na+ 143 mmol/l

K+ 5.3 mmol/l

Creatinine 195 μmol/l  Which of the following ECG features is most characteristic of moderate to severe hypothermia?Long QT interval Short PR interval 2nd degree heart block Complete heart block J waves

Whilst varying degrees of heart block may be seen in association with hypothermia, J waves are said to be most characteristic of moderate to severe hypothermia. J waves are best seen in the left chest leads and are described as a dome or hump in the terminal portion of the QRS complex. The size of the J wave is correlated with the degree of hypothermia. Slow atrial fibrillation may also be seen as core temperature falls. Death from ventricular arrhythmias is common in moderate to severe hypothermia.

A 70-year-old woman presents to the pre-operative orthopaedic clinic prior to hip replacement. She has suffered a myocardial infarction 4 years earlier and is managed with aspirin 75mg daily, ramipril 10mg daily and

45

Page 46: Cardiology question

atorvastatin 40mg daily. There is no history of angina but she is only able to walk around 50 yards. On examination she looks well, her BP is 145/80 mmHg with a pulse of 75/minute.

Bloods; Hb 14.0 g/dl

WCC 5.9 x109/l

PLT 180 x109/l

Na+ 140 mmol/l

K+ 5.0 mmol/l

Creatinine 130 µmol/l  Which of the following is the most appropriate investigation to assess her suitability for surgery from the point of view of her cardiovascular status?12-lead ECG Treadmill stress test Echocardiogram Dobutamine stress echo Cardiac angiography Dobutamine stress echo simulates the effect of exercise on the heart in patients who are unable to undertake a stress test. Dobutamine is given via IV infusion, and ECG monitoring with Echocardiography is undertaken both at rest and at the point of maximal stimulation. Patients are recommended to discontinue beta-blockade if possible for around 3 days prior to the procedure, as the negatively inotropic and chronotropic effects of beta blockade can blunt the effects of the dobutamine infusion.

A 45-year-old man was diagnosed with new onset AF after visiting his GP complaining of palpitations. An ECG confirmed atrial fibrillation with a ventricular rate of 85/minute, and an ECHO did not reveal any significant structural heart disease. On advice of the hospital he was given low molecular weight heparin and stabilised on warfarin, with an INR of 2.5.

You arrange for him to be cardioverted a few weeks later and the procedure is successful.

For how long is it recommended to continue his warfarin therapy according to current guidelines?For life For 1 week For 72 hours For four weeks For 6 months

Guidelines published in 2006 recommend warfarinisation for at least 3 weeks pre and for 4 weeks post cardioversion, aiming for a target INR of 2.5, although this can be allowed to drift up to an INR of 3 a few days prior to the procedure to minimise any risks of cancellation due to inadequate anti-coagulation. The period of 4 weeks post procedure is recommended because there is a high relapse rate in the first few weeks.

You review a 26-year-old woman who attends the cardiology clinic with her husband. They wish to start a family, but they have been referred by the GP as he is worried that she has a history of heart disease.

Which of the following cardiovascular conditions is an absolute contra-indication to pregnancy?Mitral valve prolapse Previous repaired patent ductus arteriosus Atrial septal defect Primary pulmonary hypertension

46

Page 47: Cardiology question

Bicuspid aortic valve

Whilst increased monitoring is recommended in patients with mitral valve prolapse or congenital bicuspid aortic valve, neither are absolute contraindications to pregnancy. Small atrial septal defects often remain undetected for many years, and previously repaired patent ductus arteriosis should not impair pregnancy. Primary pulmonary hypertension rapidly worsens in pregnancy however, and patients are advised not to get pregnant.

A 58-year-old man with multiple dental problems presents to the Emergency department. Apart from an abscess on his toe for which he has been receiving flucloxacillin he has been relatively well. On examination he has splinter haemorrhages and looks anaemic. You detect an aortic systolic murmur. Echocardiogram is suggestive of aortic valve endocarditis and blood cultures confirm Streptococcus viridans.

In addition to IV benzylpenicillin which antibiotic would you prescribe?Ceftriaxone Gentamicin Azithromycin Vancomycin Ciprofloxacin

The standard regime for suspected viridans endocarditis would be benzylpenicillin IV together with gentamycin 1mg/kg/day. Ceftriaxone is an alternative in stable patients and has the advantage of being given once per day. In patients who are penicillin allergic vancomycin is a viable alternative. The most likely route for his infection is via dental infection. As long as S viridans endocarditis is appropriately managed, cure rates approach 98%.

A 23-year-old woman presents to the GP complaining of palpitations. She says these are rapid and when she gets them she feels light headed and sick. They tend to come on without warning, but have occurred when she has been out dancing with friends, and after a game of squash. On examination she looks well; her BMI is 21, pulse 70/min regular, BP 122/70 mmHg.

Bloods; Hb 13.1 g/dl

WCC 5.4 x109/l

PLT 251 x109/l

Na+ 139 mmol/l

K+ 4.0 mmol/l

Creatinine 75 μmol/l  Which of the following investigations is most likely to help with the diagnosis?Tilt table test Continuous loop recorder 24hr holter monitor 3 day holter monitor 12 lead ECG  

Whilst a 12 lead ECG may reveal evidence of structural heart disease leading to changes in the resting ECG, or an aberrant pathway such as that found in Wolf-Parkinson-White, more often than not it will be normal. A holter monitor may also be in place during a period where no palpitations occur. In contrast, a continuous loop recorder can be activated by the patient during symptoms and therefore carries the greatest chance of recording the arrhythmia.

47

Page 48: Cardiology question

You review a 28-year-old woman with palpitations. On examination you suspect that there is splitting of the first heart sound. Her BP is 123/80 mmHg, P 70/min regular, and her chest is clear, there are no other cardiovascular findings.

You arrange a 12 lead ECG

Which part of the ECG is most closely associated with the first heart sound?P Wave T Wave S Wave R Wave U wave

Splitting of the first heart sound may occur in atrioventricular septal defects or in conditions such as Ebstein’s anomaly. The first heart sound occurs most closely in association with the R wave and in patients with AVSD there is usually left axis deviation, prominent P waves and prolongation of the P-R interval. Echocardiography and cardiac catheterisation follow to determine the extent of the defect.

A 58-year-old man with a history of hypertension managed with ramipril 10mg daily and 40 pack years of cigarette smoking presents to the Emergency department after a collapse at work. Neurological examination reveals a left sided hemiplegia.

Investigations; Hb 13.8 g/dl

WCC 5.4 x109/l

PLT 192 x109/l

Na+ 139 mmol/l

K+ 4.9 mmol/l

Creatinine 149 µmol/l

CT head –No intra-cerebral haemorrhage identified  What is the time limit after presentation during within which thrombolysis should be administered?1hr 3hrs 4 1/2 hrs 6hrs 12hrs

Although 3 hours was the initial time limit recommended in 2004 guidelines from the RCP, the newer SIGN guidelines (Dec 2008) have superceded these with the proposed limit of 4 and a half hour. This takes into account all currently available data on the risk-benefit ratio of treatment within this timeframe. Despite an increase in haemorrhagic stroke, the group receiving thrombolysis in the NINDs study had a lower rate of death or severe disability. Both the SIGN and RCP guidelines recommend that thrombolysis for stroke should only be undertaken in a specialist stroke thrombolysis unit.

A 62-year-old man with two previous myocardial infarctions and a history of LVF controlled with ramipril and furosemide presents to his GP with palpitations. On examination his BP is 120/72 mmHg, pulse 85/min AF, with bibasal crackles consistent with heart failure.

Investigations; Hb 12.1 g/dl

WCC 5.4 x109/l

48

Page 49: Cardiology question

PLT 234 x109/l

Na+ 140 mmol/l

K+ 5.0 mmol/l

Creatinine 130 μmol/l

ECG – Anterior Q waves, atrial fibrillation, rate 82 BPM

ECHO - Dilated left atrium and left ventricle  Which of the following would be the most appropriate agent to control his AF?Diltiazem Sotalol Amiodarone Digoxin Verapamil

Verapamil, diltiazem and sotalol are all to a greater or lesser extent negatively inotropic and may worsen cardiac failure. Amiodarone is useful for chemical cardioversion and as such is not the best choice here. Digoxin is less useful for rate control in AF than calcium antagonists or beta-blockers, but is the most appropriate choice here as it does improve symptoms in patients with cardiac failure, and given the enlarged left atrium, successful cardioversion is unlikely.

A 54-year-old woman with a history of atrial fibrillation presents with left face and arm weakness consistent with a stroke. On examination her BP is 162/82 mmHg, with a pulse of 85/min, irregular.

Investigations; Hb 12.1 g/dl

WCC 5.4 x109/l

PLT 175 x109/l

Na+ 140 mmol/l

K+ 5.0 mmol/l

Creatinine 105 µmol/l

ECG – Atrial fibrillation, evidence of previous inferior myocardial infarction

CT head – No evidence of intracerebral haemorrhage  6hrs post stroke you are considering anti-coagulation or anti-platelet therapy, which of the following would be most appropriate?Full IV heparinisation LMW heparin and commence warfarin treatment Alteplase Aspirin Streptokinase  

This patient is outside the window of the NINDS study within which thrombolysis is recommended. Meta-analysis suggested strong evidence of benefit up to 3hrs, and guidelines now support use of thrombolysis up to 4hrs after the onset of symptoms. Additionally, early anti-coagulation with heparin has been shown to increase the risk of intra-cerebral haemorrhage, without having a significant impact on the risk of long-term disability or death. As such, commencement of aspirin is the most appropriate option, with anti-coagulation at a later stage.

49

Page 50: Cardiology question

A 30-year-old man is being investigated for hypertension. A combination of BPs estimated by colour flow Doppler and measured values are listed below. Observed BPs LV 200/10 mmHg

Ascending aorta 200/70 mmHg

Right arm 190/70 mmHg

Right femoral artery 110/70 mmHg  Which of the following is the most likely diagnosis?Coarctation of aorta Left subclavian artery stenosis Aortic regurgitation Aortic stenosis HOCM

This picture would be typical of coarctation distal to the origin of the right subclavian artery. Cardiac catheterisation is the next most appropriate step in further characterising the pressure gradient across the coarctation, +/- MRI scanning. Prognosis after coarctation is dependent on speed of diagnosis. In those diagnosed after the age of 35 years, survival to age 50 is only 20%, whereas diagnosis and treatment as a child is associated with a survival above 90%.

A 53-year-old patient who has had chemotherapy for metastatic breast cancer 6 months earlier comes to the clinic complaining of shortness of breath on exertion. Her BP is 125/78 mmHg, her pulse is 94/min and her apex beat is displaced to the anterior axillary line.

Investigations Hb 11.9 g/dl

WCC 5.0 x109/l

PLT 190 x109/l

Na+ 140 mmol/l

K+ 4.5 mmol/l

Creatinine 160 μmol/l  CXR – Cardiomegaly, increased shadowing consistent with mild pulmonary oedema at both bases

Which chemotherapeutic agent is most likely to be responsible for this patient's symptoms?Doxorubicin Docetaxel Cisplatin Bleomycin Carbiplatin

Doxorubicin can be associated with cardiac failure, reduced left ventricular ejection fraction and tachyarrhythmias. Docetaxel may also be associated with arrhythmias and cardiac failure, although the likelihood is less than with doxorubicin. Platinum based chemotherapies are associated with nerve damage, and bleomycin with interstitial pneumonitis.

A 24-year-old man from a travelling family who has shunned regular medical follow up comes to the clinic complaining of shortness of breath and chest pain. You review his catheterisation results. Pressure RV 110/0 mmHg

Pressure LV 90/0 mmHg

LV oxygen saturation 88%

50

Page 51: Cardiology question

  Given the likely clinical diagnosis, which of the following is the most likely finding on clinical examination?A diastolic murmur Persistent hypoxia despite maximal oxygen therapy Tapping apex beat Narrow tented P waves on ECG Decreased pulmonary vasculature on CXR

This man has a right to left shunt, with right ventricular pressure greater than left. There is clear mixing of deoxygenated and oxygenated blood within the left ventricle as evidenced by the LV oxygen saturation of 88%. The most likely diagnosis is a long-standing VSD. The typical murmur seen is a holosystolic murmur, and the apex beat is usually displaced. P waves are broad and notched on the ECG, a sign typical of that seen with left atrial overload due to pulmonary hypertension. Of course, because of pulmonary hypertension, increased pulmonary vasculature markings are normally seen on the CXR.

A 62-year-old woman is admitted having collapsed at her local supermarket complaining of palpitations. On examination she is very unwell with a BP of 90/50 mmHg and very rapid palpitations.

Investigations;ECG – Ventricular tachycardia with moving axis – torsade de pointes

Which of the following drugs is not associated with this arrhythmia?Sotalol Verapamil Flecainide Digoxin Risperidone

Verapamil leads to a reduction in the risk of torsade de pointes (also called torsades de pointes) by reducing left ventricular transmural dispersion of repolarisation, and suppresses after depolarisations. Risperidone, sotalol and flecainide may all lead to QT prolongation and precipitate development of torsade de pointes. Digoxin in toxic levels may also be associated with ventricular arrhythmias. Acute treatment of torsade involves correction of electrolyte abnormalities such as hypokalaemia and hypomagnesemia, and removal of QT prolonging agents. Short acting beta-blockade is the mainstay of pharmacological intervention.

A 60-year-old woman with a long history of manic depressive psychosis managed with lithium therapy is sent to see you for review. She has a BP of 152/93 mmHg, and the GP is keen to commence anti-hypertensive therapy.

Investigations Hb 12.3 g/dl

WCC 5.4 x109/l

PLT 195 x109/l

Na+ 143 mmol/l

K+ 4.0 mmol/l

Creatinine 145 μmol/l

Total cholesterol 5.9 mmol/l

HDL 0.8 mmol/l  Which antihypertensive would be most appropriate for her to start?Ramipril Valsartan

51

Page 52: Cardiology question

Indapamide Amlodipine Atenolol

ACE inhibitors, angiotensin II receptor blockers both lead to increases in lithium concentration and can lead to lithium toxicity. A paradoxical anti-diuretic effect exists when thiazides such as indapamide are combined with lithium, leading to salt and water retention and increased lithium concentration. Calcium antagonists may lead to neurotoxicity when they are combined with lithium. This leaves atenolol as the available option which we are left with as, whilst it is not first choice in hypertension guidelines, it represents the safest option here.

A 42-year-old patient who has a history of paroxysmal AF has been treated with warfarin. The AF has now resolved after successful DC cardioversion.

Investigations; Hb 13.1 g/dl

WCC 4.9 x109/l

PLT 294 x109/l

Na+ 139 mmol/l

K+ 4.8 mmol/l

Creatinine 100 μmol/l

TSH 2.1 U/l  ECHO – Normal sized left atrium, no significant valvular disease

For how long should the warfarin be continued?4 weeks 6 months 1 year 3 years Stop with immediate effect

This patient has benign findings on echocardiography, with normal left atrial size; it is therefore likely that he has a reasonable chance of remaining in sinus rhythm. NICE guidelines on the management of atrial fibrillation, most recently re-issued in 2006, recommend continuing warfarin therapy for a minimum of 4 weeks. Where the risk of recurrence is high, or there are multiple failed cardioversions, then long-term warfarin therapy is advised.

A 32-year-old man presents to the clinic with shortness of breath, which is particularly bad when he goes jogging. He has recently increased his exercise to try and reduce his weight. On a couple of occasions he has also noticed some chest discomfort which has caused him to stop exercising. On examination his BP is 150/88 mmHg, and he has a double apical impulse. On auscultation there is a harsh mid systolic murmur which is loudest between the apex and the left sternal border.

Investigations; Hb 13.0 g/dl

WCC 4.8 x109/l

PLT 199 x109/l

Na+ 140 mmol/l

K+ 5.0 mmol/l

Creatinine 100 μmol/l

52

Page 53: Cardiology question

  ECG LVH and widespread Q waves

Which of the following is most directly correlated with increased risk of sudden death?Increased left ventricular outflow tract gradient Presence of mitral regurgitation Degree of left ventricular hypertrophy Asymmetrical septal hypertrophy Systolic anterior motion

A number of studies have attempted to examine potential correlations between clinical features of HOCM and increased risk of sudden cardiac death. Of the options given, degree of left ventricular hypertrophy appears to be most strongly linked to increased risk of sudden death. Increased outflow tract gradient appears to be related to symptoms like SOB and angina, and myomectomy taking tissue from the interventricular septum appears to improve these symptoms. Patients with HOCM usually die from arrhythmias, and previous VT is thus strongly predictive of the risk of sudden death.

A 42-year-old man with the features of congenital myotonic dystrophy comes to see you for review. He has suffered from mild intellectual impairment, frontal balding typical of the disease and increasing muscle weakness with increased muscle tone over the past few years. Most recently he has suffered from a number of episodes of syncope. On examination his BP is 129/70 mmHg, his pulse 55 BPM, there are no other significant findings on cardiovascular examination.

What ECG findings might you most commonly expect to see in this case?Short PR interval PR prolongation Long QT syndrome Bifasicular block Left bundle branch block

PR prolongation is the commonest feature seen in association with congenital myotonic dystrophy. Varying conduction defects including right bundle branch block, left bundle branch block and bifasicular block may be seen. The syncopes seen here may be related to periods of complete heart block, and a 72hr holter monitor would be the next most logical investigation in an attempt to capture these. The other possibility would be short runs of VT, as myotonic dystrophy also increases the risk of paroxysms of VT, again, it would be hoped that these would be captured on a holter monitor.

A 50-year-old man with no previous cardiovascular history comes to the Emergency room after referral from his GP. He attended the GP surgery with palpitations, which were extremely rapid and irregular. It is now 9am and he tells you the palpitations began the previous morning after a heavy drinking session with a friend from work. There is no history of smoking, cardiovascular disease or previous myocardial infarction, he plays squash twice per week and cycles to work. On examination his BP is 125/77 mmHg, his pulse is 140/min, irregular. He is not in cardiac failure. Investigations; Hb 13.1 g/dl

WCC 4.9 x109/l

PLT 210 x109/l

Na+ 139 mmol/l

K+ 4.7 mmol/l

Creatinine 120 μmol/l

CXR No cardiomegaly, no LVF

ECG Fast atrial fibrillation, no Q waves

Which of the following is the most appropriate therapy to chemically cardiovert him?Adenosine

53

Page 54: Cardiology question

Bisoprolol Digoxin Flecainide Verapamil

In the UK the commonest drugs used for cardioversion of atrial fibrillation are flecainide and amiodarone. Successful cardioversion is reported in up to 90% of patients given IV flecainide. It should be avoided in patients with a previous ischaemic cardiovascular history as the CAST trial suggested that mortality was increased in patients given flecanide post myocardial infarction. Digoxin does not cardiovert, and bisoprolol and verapamil are more usually employed to maintain sinus rhythm after successful electrical cardioversion. As we have a very clear history of duration of AF and no evidence of structural heart disease, successful cardioversion is likely to be possible.

An 18-year-old man comes to the Emergency room because he has suffered a severe syncopal attack whilst playing a game of squash. His opponent tells you that he collapsed and took a few minutes to recover. Apparently this was the second episode, the first having occurred after a strenuous period of exercise at the swimming pool. Of note is the fact that his father died of a cardiac arrest at the age of 32. On examination he looks fit, his BP is 132/78 mmHg, his pulse is 70/min, sinus rhythm.

Investigations; Hb 12.8 g/dl

WCC 5.0 x109/l

PLT 182 x109/l

Na+ 139 mmol/l

K+ 4.8 mmol/l

Creatinine 120 μmol/l

ECG Sinus rhythm but QT interval 0.51s

24hr tape paroxysmal AF on 2 occasions

Which of the following agents should be given for rhythm control in this case?Adenosine Flecainide Verapamil Amiodarone Metoprolol

Beta blocking agents are the drugs of choice for rhythm control in long QT syndrome. They decrease conduction through the AV node and have negatively inotropic and chronotropic effects. If patients continue to have rhythm disturbances on beta blockade then cervical sympathectomy is one treatment option, or they may be referred for Implantable cardioverter defibrillator. Lifestyle changes such as avoiding competitive or particularly intensive sporting activity may also be recommended. Other agents, particularly flecainide or amiodarone lengthen the cardiac action potential and may increase the risk of torsades de pointes VT.

An 18-year-old student is admitted to the Emergency room after a collapse in a night club. He has no recollection of the incident, was assisted by his friends and had begun to regain consciousness by the time the ambulance had arrived. On direct questioning in the Emergency room he admits to 2 previous syncopal episodes. He denies elicit drug use. On examination his BP is 123/72 mmHg, his pulse is 72 regular.

Investigations; Hb 13.2 g/dl

WCC 5.3 x109/l

54

Page 55: Cardiology question

PLT 199 x109/l

Na 142 mmol/l

K 4.6 mmol/l

Creatinine 90 μmol/l

ECG Sinus rhythm, QT interval 0.52s

A defect in which ion channel is the most likely cause of his symptoms?Magnesium Sodium Potassium Chloride Calcium

LQT1, 2 and 3 mutations account for 45%,45% and 7% of cases of long QT syndrome respectively. Both LQT1 and 2 mutations are associated with defective potassium transport, leading to a decrease in potassium outflow and more prolonged depolarisation. LQT8 is associated with defective calcium channel transport; often patients also have associated congenital heart disease and behavioural disorders. LQT3 mutation is associated with a gain of function mutation in sodium channels.

A 72-year-old man who visited his GP suffering from an infection 2 days earlier is admitted to the Emergency room after suffering a collapse at the supermarket. His wife tells you that he suffered from a myocardial infarction some 6 years ago, but has otherwise been relatively well, taking aspirin, ramipril and atorvastatin as regular medications. She doesn’t know the type of antibiotics he has been taking. On examination his BP is 120/71 mmHg and he is drowsy. His pulse is 70/min and regular.

Investigations; Hb 13.1 g/dl

WCC 5.1 x109/l

PLT 232 x109/l

Na+ 140 mmol/l

K+ 4.2 mmol/l

Creatinine 123 μmol/l

12 lead ECG sinus rhythm, QT interval 0.5s

Short runs of torsades seen on the monitor

Which of the following is the most likely causative antibiotic?Oxytetracycline Metformin Co-amoxyclav Cephalexin Clarithromycin

Both macrolides such as erythromycin, and clarithromycin, and quinolones such as ciprofloxacin and olfloxacin may lead to QT prolongation. The problem may be exacerbated by co-administration with CYP-P450 inhibitors such as ketoconazole. Initial management involves withdrawal of the potential offending agent and electrolyte assay to exclude potential exacerbating factors such as hypomagnesaemia.

A 55 year-old man with a history of mitral regurgitation and atrial fibrillation is warfarinised. His INR is therapeutic at 2.0. He needs to undergo pre-planned tooth extraction under local anaesthesia.

How would you manage him prior to the procedure?Stop warfarin for 2 days

55

Page 56: Cardiology question

Stop warfarin, start LMWH Stop warfarin, start unfractionated heparin Stop warfarin start aspirin Maintain warfarin at the therapeutic dose

For patients on short term warfarin treatment it is recommended that patients wait for treatment until after they have discontinued their period of warfarin therapy. For those on long-term warfarin therapy, British Haematological Society guidelines suggest that as long as the INR is not above 2, the procedure may take place in the standard way. UK Medicines Information (UKMI) recommendations are more relaxed, suggesting that dental procedures may take place as long as the INR is less than 4.0.

A 67-year-old man attends the cardiology clinic. He has been suffering some angina-type chest pain on going out in the cold air and is worried that he might have coronary artery disease. There is a past medical history of smoking 20 cigarettes per day, and hypertension which is managed with ramipril 10mg daily. His GP has sent an ECG which appears to show that he is in left bundle branch block.

What would you expect to hear on auscultation?Loud first heart sound, reversed splitting of the 2nd heart sound Soft first heart sound, fixed splitting of the 2nd heart sound Soft first heart sound, reversed splitting of the 2nd heart sound Soft first heart sound, normal 2nd heart sound Loud first heart sound, normal 2nd heart sound

Left bundle branch block results in the left ventricle depolarising from cell to cell conduction via the right ventricle, rather than via the normal pathway. This results in reversed splitting of the second heart sound, in other words split in expiration and single in inspiration. Additionally the first heart sound tends to be softer than usual. Left bundle branch block in this age group is most likely to be ischaemic in origin, hence his chest pain almost certainly warrants further investigation, especially given his history of smoking and hypertension.

You are looking at drawing up guidelines for appropriate use of IIb 3a inhibitors within your hospital.

Looking at available evidence, what is the most appropriate indication for using these therapies?In a patient presenting with chest pain without ECG changes A patient with chest pain, a positive troponin and awaiting angiography In conjunction with thrombolysis in an MI with ST segment elevation In a thrombolysed patient suffering continuing chest pain In a patient with T wave inversion who is pain free

This patient population has been examined in clinical studies with and without background clopidogrel therapy. Even on top of clopidogrel therapy a 35% reduction was seen in a combined endpoint of death, target vessel revascularisation and MI at 30 days. Other studies involving patients not undergoing PCI were not positive, and in ST segment MI alternative anti-thrombotic regimens, for instance using fondaparinux may be more appropriate.

You are working in the chemical pathology lab and receive a sample request for analysis of B-type natriuretic peptide (BNP). You don’t have any clinical details on the form apart from “chest pain”. You plan to ring the SHO who requested the test for further details.

In which of the following situations is BNP most likely to be normal?Unstable angina Pericarditis Pulmonary embolus  

56

Page 57: Cardiology question

Acute myocardial infarction Acute mitral valve rupture

BNP is secreted in response to raised intra-cardiac pressures, primarily due to volume distension, and leads to increased sodium excretion and decreased systemic vascular resistance. Both acute myocardial infarction and acute mitral valve rupture may result in volume distension, leading to elevated levels of BNP. Constrictive pericarditis may also lead to raised intracavity pressures and hence elevated BNP. Large pulmonary embolus produces raised right sided cardiac pressures and thus again may lead to elevated BNP. Thus the only other remaining option is unstable angina.

A 62-year-old woman suddenly deteriorates 2 days after receiving TPA for an acute myocardial infarction. She complained of severe shortness of breath during the course of the afternoon and when the nurses examined her, her O2 saturation was only 91% on oxygen delivered via re-breather. On examination her BP is 105/70 mmHg, with a pulse of 105/min regular. She has an apical systolic murmur and marked left ventricular failure.

Which of the following is the most likely cause?Acute VSD Acute ASD Pericardial tamponade LV wall rupture Papillary muscle rupture

The posteromedial papillary muscle is twice as likely to rupture as the anterolateral one because the posteromedial papillary muscle is supplied by the right coronary artery only, whereas the anterolateral papillary muscle receives supply from both the left anterior descending and left circumflex arteries. The murmur is consistent with mitral regurgitation which in turn leads to acute left ventricular failure. Management centres on decreasing afterload in an attempt to stabilise patients before undergoing valvular surgery; sodium nitroprusside is the usual therapy of choice where blood pressure allows.

A 70-year-old man presents with severe tearing back and chest pain which came on very suddenly. He has a past medical history of hypertension for which he takes ramipril 10mg daily, amlodipine 5mg, and he smokes 30 cigarettes per day. On examination he is in severe pain, his BP is 155/85 mmHg, he has bilateral upgoing plantars and 4/5 weakness affecting left ankle dorsiflexion. He appears to have a pericardial rub.

Which of the following features is most suggestive of dissecting aortic aneurysm?The pattern of pain described Hypertension Bilateral upgoing plantars Left lower limb signs Pericardial rub

The acute onset of severe tearing back and chest pain is very typical of dissecting aortic aneurysm. It is impossible to tell whether his neurological signs seen are new or old, and a pericardial rub or hypertension are more likely to be associated with other causes of chest pain. Upper limb neurological signs are more likely to be associated with thoracic aortic dissection, lower limb signs may be commoner in anterior spinal artery dissection or thrombosis. Painless aortic dissection only occurs in around 10% of patients, and is more common in patients who have connective tissue disorders such as Marfan’s.

A 72-year-old man was admitted with an acute anterior myocardial infarction. He has chronic renal impairment, with a recent creatininerecorded at 148 μmol/l. Medication included ramipril, atorvastatin and indapamide for the treatment of hypertension. He was taken straight to the angiography suite where he received stenting of a left main stem stenosis. You are asked to see him about 30hrs after as the nurses feel he is deteriorating. On examination his BP is 149/84 mmHg, his pulse is 75/min and regular. His legs look dusky in colour, particularly his right big toe which looks blue in colour. He has splinter haemorrhages affecting toenails on both feet. There

57

Page 58: Cardiology question

is a loud left femoral bruit. Investigations; Hb 13.2 g/dl

WCC 5.0 x109/l

PLT 190 x109/l

Na+ 141 mmol/l

K+ 5.9 mmol/l

Creatinine 630 μmol/l

Urine blood ++, protein +  Which of the following is the most likely diagnosis?Renal vein thrombosis  Acute tubular necrosis Renal artery stenosis Cholesterol embolism Femoral artery embolism

Risk factors for cholesterol embolism after coronary artery instrumentation include increased age (>60 years), hypertension, cerebral vascular disease and aorto-iliac arterial disease. Further vascular procedures, anti-coagulant and thrombolytic therapies are not of value in the management of the condition. Patients should be dialysed during the acute period as they may recover a limited amount of renal function. Unfortunately the prognosis of cholesterol embolism is very poor: where multiple organs are involved mortality may approach 90% at 3 months.

A 21-year-old woman presents to the clinic with symptoms of increased shortness of breath and decreased exercise tolerance. She used to be a keen hockey player when at school but is now virtually unable to even walk to the bus stop without becoming short of breath. On examination she looks tired and slightly short of breath at rest. Her BP is elevated at 145/92 mmHg.

Investigations;

Echocardiogram – increased right atrial size, elevated right arterial pressure by Doppler

Cardiac catheterization;

O2 saturation SVC 74% O2 saturation RA 82% O2 saturation RV 82% O2 saturation LA 91% O2 saturation LV 91%

Which of the following is the most likely diagnosis?Ostium primum atrial septal defect Secundum atrial septal defect Patent ductus arteriosus Pulmonary stenosis Tricuspid regurgitation

Secundum defects may often be diagnosed in patients entering early adulthood. The difference in saturations between the SVC and the right ventricle indicates that there is a left to right shunt of oxygenated blood. Patients with secundum defects tend to be slim and not to suffer from cyanosis. Therapy of choice is via delivery of a catheter device to close the defect, or surgical closure if the defect is particularly large.

58

Page 59: Cardiology question

A 52-year-old man is admitted to the intensive therapy unit with left ventricular failure post- myocardial infarction. Despite prompt activity including angioplasty within a few minutes of the onset of chest pain, his systolic BP on admission to the unit was only 80 mmHg, with a pulse of 105/min. Auscultation of the chest revealed crackles up to the mid zones on both sides consistent with cardiac failure. The team decide to insert an intra-aortic balloon pump timed to coincide with the dicrotic notch.

What does the dicrotic notch refer to?Aortic valve opening Aortic valve closure Mitral valve opening Mitral valve closure Pulmonary valve closure

The dicrotic notch refers to a secondary upstroke in the downward part of the pulse wave which corresponds with closure of the aortic valve. Intra-aortic balloon pumps deflate during systole which then increases forward blood flow because of the reduction in afterload. The pump then re-inflates during diastole, increasing the blood flow to coronary arteries.

A 72-year-old woman is admitted for assessment after two episodes of collapse over the past few months. She has been managed by her GP for many years for hypertension and is currently treated with indapamide and amlodipine. On examination she looks a little thin. Her BP is 175/125 mmHg. Auscultation of the chest reveals an ejection systolic murmur. She has minor crackles at both lung bases.

Investigations; Hb 13.0 g/dl

WCC 5.3 x109/l

PLT 194 x109/l

Na+ 140 mmol/l

K+ 4.2 mmol/l

Creatinine 145 μmol/l

Cardiac catheterisation gradient of 50mmHg across the valve

Which of the following would most influence your decision to refer this patient for valve replacement?Gradient of 50 mmHg Presence of left ventricular hypertrophy Presence of symptoms Presence or absence of valvular calcification Her relatively young age

Whilst in gradient terms her valvular stenosis is on the cusp of the severe category, elective valve replacement is generally not recommended in the absence of symptoms. If there is co-existent coronary artery disease, then elective valve replacement may be undertaken at the same time as CABG. Elevated age is not a barrier to aortic valve replacement, functional status prior to surgery is much more important. In selected patients who are unfit to undergo surgery, balloon valvuloplasty may provide symptomatic relief for a period of 6-12 months.

A 73-year-old woman is admitted for pacemaker insertion because of a number of syncopes and periods of complete heart block identified on 72hr ECG. She receives a DDDR pacemaker.

What does the R stand for?Rate limiting Rate modulated

59

Page 60: Cardiology question

Repolarising Rate enhancing Rate reducing

DDDR stands for dual chamber paced, dual chamber sensed, dual response, rate modulated device. In other words, the activity of the pacemaker is varied according to the background heart rate. Dual chamber pacing devices are less likely than ventricular pacing only devices, which are associated with increased risk of AV dysyncrony.

A 32-year-old woman is admitted in an unconscious state after an overdose of a large number of amitriptyline tablets. It is thought that she took them between 7 and 8pm and was not found by her partner until he returned from a bar some 3hrs later. When you get to see her she has already been intubated by the Emergency department consultant. Her BP is 100/70 mmHg and she has a sinus tachycardia of 100 BPM. While you are watching the monitor you can see she is suffering from short unsustained runs of ventricular tachycardia.

Investigations; pH 7.29

pO2 8.1 kPa

pCO2 4.9 kPa

HCO3- 13 mmol/l

  Which of the following is the most appropriate way to initially manage the short runs of VT?Normal saline infusion Magnesium infusion Amiodarone infusion Adenosine bolus IV Sodium bicarbonate

Whilst magnesium may be a useful option to control VT in cases of severe tricyclic antidepressant (TCA) overdose, IV sodium bicarbonate is the initial therapy of choice. Increasing the pH to the range of 7.45-7.55 has been shown to decrease the QRS interval, stabilise arrhythmias and increase blood pressure. Other case series state that phenytoin may be useful in correcting conduction defects in this patient population. Conventional class IA, IC, II and III anti-arrhythmic agents should however be avoided.

A 54-year-old man with a history of smoking and hypertension presents to the Emergency room with central crushing chest pain, nausea and sweating. On examination his BP is 104/70 mmHg, his pulse 85/min regular and he looks pale, grey and sweaty. There are no murmurs on auscultation but he has crackles at both lung bases consistent with heart failure.

Investigations; Hb 12.8 g/dl

WCC 5.9 x109/l

PLT 190 x109/l

Na+ 141 mmol/l

K+ 5.0 mmol/l

Creatinine 110 µmol/l

ECG ST elevation V1-V4, ST depression II, III and aVL  Which of the following is the most likely finding on angiography?Hypothermia promotes pulmonary vasoconstriction70% stenosis of left anterior descending artery Total occlusion of left anterior descending artery

60

Page 61: Cardiology question

70% stenosis of left circumflex artery  Total occlusion of left circumflex artery

This man is suffering from an ST elevation anterior myocardial infarction and most likely has an occlusion of his left anterior descending artery due to plaque rupture. Management of choice is primary PCI, as early as possible after the onset of chest pain. Studies have now confirmed that primary PCI is superior to thrombolysis with rates of arterial patency in more than 90% of procedures, and lower rates of bleeding complications. Left circumflex or right coronary artery occlusions would be expected to result in either lateral or inferior ECG changes.

A 50-year-old man presents to the cardiology clinic for review. His brother died suddenly of a cardiac arrest while playing Sunday league soccer, and was found on post-mortem to have an underlying diagnosis of hypertrophic obstructive cardiomyopathy (HOCM).

Which of the following is the most appropriate way to screen the patient?Exercise ECG Electrocardiography studies Trans-oesophageal ECHO Trans-thoracic ECHO Resting 12 lead ECG

Two dimensional echocardiography is diagnostic for HOCM, with TOE delivering the best views. Elevated flow velocity across the left ventricular outflow tract is seen, diastolic dysfunction with reduced LV compliance, systolic anterior motion of the anterior mitral valve leaflet, and asymmetrical septal hypertrophy are also seen. Radionucleotide imaging may show reversible perfusion defects, but these are not diagnostic of the condition. Cardiac catheterization follows ECHO, to assess the degree of left ventricular outflow tract obstruction, LV anatomy and coronary artery anatomy.

61


Recommended